The Essential Guide to the ACT - My College Options

3 downloads 84 Views 2MB Size Report
(Answer sheets for taking the Practice Test can be found at the back of the Guide. ) ... cluded with this guide is a fun, easy-to-use practice test in the form of an ACT  ...
The Essential Guide to the ACT Written by: Dawn Burnette Stefan France Joyce Suber Edited by: Joyce Suber



Table of Contents Introduction………………………………………………………..……………………… 5 Chapter One Essential Facts………………………………………………..…………………… 7 Chapter Two Test-Taking Tips………………………………….…………………………....….. 9 Chapter Three English Skills Review…..…………………………………………………..……… 11 Chapter Four Math Skills Review………………………………………….……………………. 41 Chapter Five Science Skills Review……………………………………….……………………. 107 Chapter Six Practice Test………………………………………………….…………………… 119 Chapter Seven Practice Test Answers (with explanations)………….…………………………..… 179 Chapter Eight Additional Resources………………………………………….….………………. 209

(Answer sheets for taking the Practice Test can be found at the back of the Guide.)







Introduction to The Essential Guide Welcome to the world of ACT test prep! Like thousands of other high school students, you are probably both excited and anxious about all of the steps that will lead you to a college that is right for you. Among those steps is taking college entrance exams, and perhaps you are wondering what you can do to make sure that you achieve your very best on these tests. Well, not to worry. Help is on the way! Overview and Goals of the ACT Prep Program Our materials are designed for students who want to increase their scoring potential or improve on previous scores on the ACT and for those who are seeking a refresher course in particular English, reading, math, science, and/or writing skills. In this Guide, you will find those things, and more. Our program is also designed to boost your confidence in taking college entrance exams by providing: • • • • • • •

vital information about the nature and construction of the ACT and about test registration; tips for mastering the art of taking standardized tests in general and the ACT specifically; formal instruction and tips for tackling test prep and taking the exam; help for increasing your reading vocabulary through our use of college level writing and the inclusion of unfamiliar words throughout the content of the Guide; the student perspective on issues related to college entrance testing; a fun way—in video game format—to review material and practice test-taking skills; and a brief list of other resources to aid you in preparing for the ACT.

In fact, in this Guide you will find a unique approach to mastering the ACT. Our approach is tailored to students who are self-motivated—those who can and will work on their own in preparing for the ACT and are not interested in using the typical “big, fat book” of 300+ pages to do so. Instead, included with this guide is a fun, easy-to-use practice test in the form of an ACT test prep game called Zero Hour Threat. Created by i.d.e.a.s. at Disney-MGM Studios, it is an interactive action game designed to increase standardized test scores as well as enhance general mathematics, science, reading, and vocabulary skills. With each correct answer, students move one step closer to decoding a virus that international criminals have set in place to infect the United States’ banking systems. You will be able to work on enhancing your ACT scores while having fun through playing a state-of-theart video game. The Zero Hour Threat CD is not a stand-alone study program. It is designed for use in conjunction with the review materials, questions, and other tools provided in The Essential Guide to the ACT. In addition to the CD game, you will find skill-building exercises, along with one practice test (in the basic format and time frame of the real test) in each content category of the ACT to provide you with simulated experience in taking the actual ACT. For each of these, we will provide the opportunity for you to score your test by providing the correct answers and the rationale behind each of those answers.



Frequently Asked Questions and Student Concerns Below is a list of some frequently asked questions and concerns raised by students regarding the ACT. • • • • • • • • • • •

Why should I take the ACT? What is the difference between the ACT and the SAT Reasoning Test? How do I register for the ACT? What subjects are covered on the ACT? How is the ACT scored? What tools may I or should I bring to the ACT test site on the day of the exam? What is the format of the exam? How can I prepare for the exam? Which method of preparation for the ACT is best? How long should I spend on each question? If I don’t know the answer to some questions, should I guess?

Each of these questions will be addressed specifically in the appropriate chapters of the Guide. You should read each chapter carefully and thoroughly in order to make the most of the material provided. You should also jot down any other questions that come to mind as you read and engage in the activities found in each chapter. If you find that you have additional questions or concerns that are not answered as you move through the Guide, you should consult the guidance or college counselor at your school or visit the ACT web site at www.act.org/index.html for further information about the test program. Optimizing Your Use of The Essential Guide to the ACT To get the most out of the Guide, we suggest that you follow the directions to the letter. You should plan to review one chapter at a time, setting aside time each day for studying the content and completing the exercises. Review the vocabulary charts in Chapter Three daily, and use the suggested strategies for increasing your reading comprehension and skill. You can, of course, play Zero Hour Threat as frequently as time permits. (Remember that getting into college depends largely on your academic performance and that success at school is a result of hard work in the classroom and completion of your homework and other assignments.) We hope that you will find our approach exciting and rewarding. After utilizing this unique test preparation method that combines work and play, you should be on your way to increasing your ACT scoring power. Using The Essential Guide to the SAT will… 1) Boost your confidence 2) Increase your potential for high scores 3) Increase your vocabulary 4) Help you master the art of test-taking



Chapter One: Essential Facts What is the ACT? The American College Testing (ACT) examination measures skill in four areas: English, reading, science, and mathematics. In addition, there is an optional writing test offered in a version of the exam called ACT Plus Writing. The ACT is designed and developed by American College Testing in keeping with standards and codes set by the National Council on Measurement in Education, The American Psychological Association, The American Educational Research Association, and The Joint Committee on Testing Practices. Students are allocated 2 hours and 55 minutes to answer the two hundred and fifteen (215) multiple-choice questions on the exam. The optional writing section requires an additional 30 minutes of testing time. Since some colleges require the writing test, you should check with the colleges to which you are applying to ascertain whether this portion of the ACT is a requirement. Each of the four test areas of the ACT (English, mathematics, reading, and science) is scored 1 to 36, with 36 being the highest possible score. These four test scores are then averaged and rounded to the nearest whole number to obtain a composite score from 1 to 36. If you take the ACT Plus Writing, your essay will be evaluated by two professional readers who will assess the overall quality of your written expression and award your essay with a subscore of 1 to 12. This subscore will be combined with your English test score (2/3 = English questions, 1/3 = essay) to arrive at a composite English test score. Your score report for the ACT Plus Writing will include both your writing score and your English score, before and after the composite English score is determined. (For details regarding the writing score, visit http://www.actstudent.org/writing/ scores/index.html.) Below is the format of the ACT exam: English (45 minutes/60 questions) (1-36 points) Measures skill in rhetoric and standard written English Math

(60 minutes/60 questions) (1-36 points) Measures skill in mathematics normally gained from instruction through 11th grade (numbers, operations, algebra/functions, geometry, trigonometry)

Reading (35 minutes/40 questions) (1-36 points) Measures skill in reading comprehension Science (35 minutes/40 questions) (1-36 points) Measures skill in reasoning, analysis, interpretation, evaluation, and problem solving related to scientific material and evidence Writing (Optional—30 minutes, in response to one writing prompt) (subscore = 2-12) Measures skill in writing normally developed through high school instruction and expected for success in completing early college writing assignments



Who Should Take the ACT? As a college entrance examination, the ACT is taken by high school juniors and seniors. Along with a record of your academic performance (the high school transcript), evidence of involvement in extra-curricular activities, recommendations, essays, and other supportive documents — the ACT is accepted by the majority of the colleges and universities in the United States as a part of the application profile. The ACT is administered five times per year between October and June; however, some states also offer the exam in late September as well. Since the exam measures mathematics skills through the third year of college preparatory study, we recommend that you take your first ACT no earlier than the spring of your junior year in high school unless you are highly advanced in your academic work. If you wish to try to improve your initial score, you should feel free to take the exam again in June or in the first semester of your senior year, prior to your college application deadlines. How do I register for the ACT and what fees apply? ACT test registration materials are available in your high school guidance or college counseling office. You will find test dates and registration deadlines posted in both of these locations as well. You can also register online at www.actstudent.org/regist/elecreg, where you will find the testing dates for the current year. At this website, you will also find high school, college, and test center codes and information about eligibility for fee waivers and special circumstances related to ACT testing. Whether you register by mail or online, it is wise to read the registration material carefully, since other fees may apply under various conditions. The basic fees for taking the ACT are outlined in the registration materials for the exam. These charges cover the administration of the test and your score report and allow you to send your scores to as many as four colleges. Fee waiver information is also available through your guidance or college counselor, and fee waiver cards can only be procured from the counselor, even for online registration. Home-schooled students must provide the local high school proof of eligibility for fee waivers. To use the card, you must register for the ACT exams according to posted regular deadlines .



Chapter Two: Test-Taking Tips In the next three chapters, we will focus on the review of skills related to the areas tested on the ACT. Along with skill review, you will find tips and strategies for tackling specific types of questions. This chapter will center on general tips for taking standardized college entrance tests. While we cannot guarantee that these tips will work for every student, you should consider them as good advice for any test-taker. Nora, a student who recently took the ACT, offers the following advice: • Dress in layers and bring a sweater or jacket. Test sites use the cafeteria, auditorium, or other spaces that are large enough to hold all of the test takers, but schools often turn off the heat or air conditioning on the weekends. It is difficult to concentrate when you are too hot or too cold. • Bring a watch. Basic time management is up to you! The proctors tell you how much time you have for each section, when to start, when to stop, and when you have 5 minutes left. You should pay attention to how long you are taking on the questions and pace yourself during the test. • Skip hard questions or use your best guess. You can usually narrow your choices down to two possible correct answers. That will give you a 50% chance of answering the questions correctly. (We will cover guessing in more detail below.) Approaches to this strategy differ when taking the SAT. • Replace, in your mind, long hard story character names like Jedidiah, Beauregard, Shaneequa with Bill or Jane. You can get lost in the names and it can draw your attention away from the question. • Read carefully! Make sure you understand the main idea of the story. Ask yourself these two questions: 1) What is the story about -- the subject? 2) How does the subject of the story relate to the questions? • Review the basic concepts — arithmetic operations and their order in solving problems, algebraic formulas, etc. • Prepare yourself by studying a few days of basic math formulas a week before the test (Example: A=bh2). • Know a bit about word problems and how to apply basic formulas to them. • Know how to solve 3-D and 2-D shape problems involving circumference, perimeter, and area. • Be prepared for lots of questions using algebra. (Example: 2x = 4 or xy = 0).



• Be aware that a few questions may include seldom-used units of measurement, such as “stones.” The key is to use “proportion.” Do not worry about the unit. (Example: If 100 bags of sand weigh 300 stones, how much would 170 bags weigh? Simply cross multiply to find “x”.) • Bring a graphing calculator. It will help in converting decimals to fractions and vice versa, with square root, etc., and will save a lot of time. • Sharpen your science skills in reading and understanding graphs, charts, tables, and other schematic forms. Review the scientific terms you have learned in your high school science courses. • Familiarize yourself with reading and understanding scientific research summaries. Be able to recognize and understand related and conflicting viewpoints when reading scientific data. • Write on the test! Underline important material and make notes that might help you answer the questions correctly. To Guess or Not to Guess? As mentioned above, the question of guessing on the responses to the ACT is a common one. While some would say that guessing is cheating, it is important to understand that there is no penalty for wrong answers on the ACT. Therefore, you should always guess if you do not know the answer to an ACT question. Before you guess, however, you should try to eliminate at least one wrong answer before choosing a response. (Eliminating two is even better!) Guessing wisely rather than randomly will increase the likelihood that your guess will yield the right answer. Some Final Thoughts Other general test-taking advice includes the following: • Get a good night’s sleep before the test. • Eat a light breakfast and take a snack and something to drink for break times. • Arrive at the test site early or on time. (You will not be allowed to take the test if you are late!) • Along with a watch and calculator, bring several sharpened #2 pencils and erasers. • Try to relax; then concentrate on following directions and answering the questions to the best of your ability. By utilizing these tips and strategies along with those offered in Chapters 3 and 4, you will increase your chances for success in taking the SAT Reasoning Test. So jump right in, review the comments provided in this guide, play the CD game, and have fun preparing for the SAT.

10

Chapter Three: English Skills Review The English section of the ACT consists mainly of multiple-choice questions that test your knowledge of grammar, conventions, and sentence structure. Only the most grammar-savvy students should face this section of the test without some grammar review. Below are some of the most common types of grammatical issues you’ll encounter on this part of the test. Subject/Verb Agreement • This is by far the test writers’ favorite type of question. • It is important that your subjects agree with your verbs. In other words, you must write he jumps instead of he jump because a singular subject (he) takes a singular verb (jumps). • Subject/verb agreement gets tricky when a prepositional phrase comes between the subject and the verb. However, you should ignore the prepositional phrase and make the verb agree with the subject. (One of the girls likes the movie.) • Note that the following pronouns are singular: each, either, neither, someone, somebody, everyone, everybody, anyone, anybody, no one, nobody. (Neither of the movies was very good.) • When two or more subjects are joined by and, you must use a plural verb. (Hiking and camping are fun.) • When two or more subjects are joined by or or nor, the verb agrees with the subject nearest the verb. (Neither the players nor the coach is late for practice.) • Amounts are usually singular, as are titles. (Two hours is a long time to wait. Great Expectations is a great book.) Try this practice question: Help for improving test-taking skills and reading skills are also available.

A. NO CHANGE B. for reading skills are C. reading skills is D. for reading skills is

The correct answer is C. The subject (Help) is singular and therefore requires a singular verb (is, not are). The prepositional phrase (for improving test-taking skills and reading skills) is meant to trick you because it’s plural. It’s also long so that by the time you reach the verb, you’ve lost track of the subject, so be careful! Adding for (choices B and D) causes the sentence to be unparallel (See Parallel Structure). Pronoun/Antecedent Agreement • This concept is very similar to subject/verb agreement and is another favorite of the test writers. • A noun that a pronoun refers to is called its antecedent. For example, in the sentence The boy rode his bike, boy is the antecedent of the pronoun his. Just as a subject has to agree with its verb, a pronoun has to agree with its antecedent.

11

• If the antecedent is singular, the pronoun must be singular; if the antecedent is plural, the pronoun must be plural. • Pronoun/antecedent agreement also gets tricky when the antecedent is followed by a prepositional phrase. However, you should ignore the prepositional phrase and make the pronoun agree with the antecedent. (One of the girls forgot her pencil.) • Note that the following pronouns are singular: each, either, neither, someone, somebody, everyone, everybody, anyone, anybody, no one, nobody. (Neither of the players remembered to tie his shoes.) • When two or more antecedents are joined by and, you must use a plural pronoun. (Katianne and Ralston took off their shoes.) • When two or more antecedents are joined by or or nor, the pronoun agrees with the antecedent closest to it. (Neither the players nor the coach brought his stopwatch.) Try this sample question: If a student wishes to be involved in the class, they must get parental consent first.

A. NO CHANGE B. he or she must C. they have to D. he or she have to

The correct answer is B. The antecedent of the pronoun is student, which is singular. Therefore, the pronoun must be singular as well (he or she rather than they). Choice D is incorrect because when two subjects are joined by or (he or she), the verb agrees with the one it’s closer to (she). Parallel Structure • When you join two or more ideas—in the form of words, phrases, or even clauses—in a sentence (usually with the help of a conjunction), the ideas you join must be parallel. In other words, the grammatical structure of each one must be the same. For example, Michelle is kind, helpful, and likes puppies is not parallel because we’re joining two adjectives (kind and helpful) and one verb (likes). To correct the sentence, you could write Michelle is kind, is helpful, and likes puppies (three verbs) or Michelle is kind, helpful, and loving (three adjectives). Try this sample question: Lucie is not only a loving character, but she is a strong character as well.

A. NO CHANGE B. character, in addition to that she is also very strong C. character, but she is also a strong character D. character but also a strong character

12

The correct answer is D. In the original sentence, an adjective and noun follow the not only part of the correlating conjunction, but a subject and verb follow the but part of the correlating conjunction. Instead, we need an adjective and noun to follow the but. Note that the correct answer is also the shortest, as is often the case. Passive Voice • Although passive voice is not grammatically incorrect, it is often considered to be stylistically unpleasant. • Passive voice consists of a to be verb (is, be, am, are, was, were, been, being) and a past participle (such as stolen, pushed, or enjoyed). • The subject of the passive voice verb is receiving the action rather than doing the action. – for example, The play was enjoyed by the audience. Although play is the subject, the object of the preposition (audience) is actually doing the enjoying. The sentence would sound much better if it were active: The audience enjoyed the play. • Not all past participles indicate passive voice, however. For example, in the sentence I have listened to that CD all day long, have listened is not passive voice. • Although there are some cases in which passive voice is acceptable, use active voice whenever possible. Try this sample question: The letter is written by Sir Andrew and Sir Toby as a trick to make a fool of Malvolio.

A. NO CHANGE B. Sir Andrew and Sir Toby write the letter C. The letter, which is written by Sir Andrew and Sir Toby, D. Sir Andrew and Sir Toby write the letter, and they do it

The correct answer is B because the subjects (Sir Andrew and Sir Toby) do the action (write). In other words, the sentence is in active voice rather than passive voice. While D is also active voice, it’s wordy. Remember, on the ACT there may be more than one correct answer, but you’re looking for the best one. Stay away from wordiness. Use of Modifiers • A modifier is a word that describes another word. • Modifiers come in three degrees of comparison: positive (tall, carefully), comparative (taller, more carefully), and superlative (tallest, most carefully). Use the positive form when you’re just describing something. (Joe is tall.) Use the comparative form when you’re comparing two things. (Joe is taller than John.) Use the superlative form when you’re comparing three or more things. (Joe is the tallest person in his class.) • Avoid using an adjective to modify a verb. (He ran quickly, not He ran quick.)

13

• When you’re comparing something with a group it belongs to, you must include the word other or the word else. (This class is more fun than any other class. Mrs. Miller is nicer than anyone else I know.) • Note that less, amount, and much refer to collective nouns (uncountable things like traffic and rain), fewer, number, and many refer to nouns you can count (like cars and rain drops). • A modifier is said to be “dangling” when it refers to a word not directly stated in the sentence. • A misplaced modifier is one that is in the wrong place in sentence. • Always put modifiers as close to the words they modify as possible. Otherwise, your sentences are unclear. Try these practice questions: 1) When you captivate the attention of your audience, you have less conduct problems.

A. NO CHANGE B. one has less C. one has fewer D. you have fewer

The correct answer is D. Since you can count conduct problems, you must use fewer instead of less. Don’t be tricked into choosing C. Changing you to one would result in a shift from second person in the first clause (you captivate) to third person (one has) in the second clause. 2) The sirens cause passing sailors to go completely insane by singing beautiful songs.

A. NO CHANGE B. Singing beautiful songs, the sirens cause passing sailors to go completely insane. C. The sirens, which are always singing beautiful songs, cause sailors to go completely insane. D. The sirens, who cause passing sailors to go completely insane, are always singing beautiful songs.

The correct answer is B because singing beautiful songs modifies the sirens. Choice A suggests that the sailors go insane by singing beautiful songs themselves! Choices C and D are wordy and awkward. Pronoun Usage We already talked about pronoun/antecedent agreement, but you must also know when to use which pronoun case (such as I versus me). • You must use nominative pronouns (such as I, he, and they) for subjects and predicate nominatives (for example, Jacob and I are studying for our math test). • You must use objective pronouns (such as me, her, and them) for direct objects, indirect ob-

14

jects, and objects of prepositions. (Will you eat lunch with Rudy and me?) • Most students have trouble deciding when to use who and when to use whom. o Who (nominative case) is for subjects and predicate nominatives. o Whom (objective case) is for direct objects, indirect objects, and objects of prepositions. Also, remember that hisself, theirselves, and themself are not words! Try this practice question: Since Mr. Key wasn’t sure who was responsible, he gave both Lauren and I detention.

A. NO CHANGE B. gave both Lauren and myself C. both gave Lauren and I D. gave both Lauren and me

The correct answer is D. You must use the objective pronoun me rather than the nominative pronoun I because Lauren and me is an indirect object. Note that the use of who in this sentence is correct because it acts as the subject of the clause who was responsible. Verb Tense • As a general rule, you should use past tense (walked) to write about something that happened in the past, present tense (walks) to write about something that is happening right now, and future tense (will walk) to write about something that hasn’t yet happened. • In a verb phrase, the tense is determined by the helping verb. For example, has walked is present perfect tense, but had walked is past perfect tense. • Avoid switching verb tense within your writing. For example, if you’re telling a story and using past tense, don’t randomly switch to present tense. Try this sample question: When Kyle comes over to my house, we started playing video games immediately.

A. NO CHANGE B. start playing C. will start playing D. started to play

The correct answer is B. Started is a past tense verb, whereas comes is present tense. Choices C and D are both wrong because C offers a future tense verb and D offers a past tense verb.

15

Verb Usage Sometimes you might use the right verb tense and correct subject/verb agreement, but you use the wrong verb or verb form. Here are a few tips regarding verbs that people sometimes misuse: • Lie, sit, and rise are all intransitive verbs and are therefore not followed by a direct object. (I like to lie under the tree.) • Lay, set, and raise are transitive and therefore are followed by a direct object. (Set the books on the table.) • Shall goes with I and we (I shall go to the movie.) Will goes with everyone else. (She will go to the movie.) • Use may when you’re referring to permission. (May I have that pencil?) • Use can when you’re referring to ability. (Can you run twelve miles?) • Avoid using the wrong form of a verb. For example, a past tense verb (spoke) does not need a helping verb, but a past participle (has spoken) does. Try this practice question: On weekends I like to lay around and watch movies.

A. NO CHANGE B. to lie around C. laying around D. to keeping lying around

The correct answer is B. Here we have to use the intransitive verb lie rather than the transitive verb lay because we don’t have a direct object. Although around may look like a direct object, it tells “where” rather than “what.” Choice D does use lying rather than laying, but it is unnecessarily wordy. Run-on Sentences • When you have two whole sentences stuck together with only a comma in between them, you have a type of run-on called a comma splice (such as I’m starving, I hope we can eat soon). • A comma is not strong enough to hold two sentences together. • You must add a coordinating conjunction (such as I’m starving, so I hope we can eat soon), change the comma to a semicolon (such as I’m starving; I hope we can eat soon), create two separate sentences (such as I’m starving. I hope we can eat soon), or reword the whole thing (such as I hope we can eat soon because I’m starving). • When you have two whole sentences (or even more than two) stuck together with nothing at all in between them, you have a run-on sentence (such as I’m starving I hope we can eat soon). • You can correct the run-on sentence the same way you correct the comma splice.

16

Try this practice question: We have two hours to get there, that’s plenty of time.

A. NO CHANGE B. there that’s plenty of time. C. there, which is plenty of time. D. there, which certainly should be plenty of time.

The correct answer is C. Choices A and B are both run-on sentences. Although D is technically not wrong, it is wordy, making C a better choice. Sentence Fragments • A sentence fragment is a sentence that isn’t complete. • In order to be complete, a sentence must be a complete thought with a subject and a verb. • Beware of sentences that start with subordinating conjunctions (like if, because, since, and although). • A clause that starts with a subordinating conjunction is dependent and cannot stand alone as a sentence, so be sure that you have another clause—an independent one—following the dependent one. Try this sample question: Although now I can say I’m special and mean it.

A. NO CHANGE B. Although, now I can say I’m special and mean it. C. Although I called myself special before, now I really mean it. D. Now I call myself special, however I didn’t mean it before.

The correct answer is C. It consists of a dependent clause (Although I called myself special before) and an independent clause (now I really mean it). Choice A consists of a dependent clause only. So does choice B. The comma after although doesn’t change that fact. Choice D is a comma splice. Wordiness • Clear, effective writing says more with fewer words. • Inexperienced writers sometimes think that they need to add extra words to their papers to make them sound better. • Your goal should be to add more content (information, examples, description, explanation, etc.) instead of to add empty words. • When in doubt on the “sentence improvement” section, choose the shorter answer. Although you won’t be correct every time, more often than not you will be.

17

Try this sample question: This is an excellent book that is extremely fun and helpful to read.

A. NO CHANGE B. This is an excellent book which is extremely fun and helpful C. This excellent book, which is extremely fun and helpful D. This helpful book is extremely fun

The correct answer is D because it is short and sweet. A and B contain unnecessary words. Choice C makes the sentence into a fragment. Word Choice (Diction) Sometimes you will have to identify words that are misused rather than grammatically incorrect. For example, a sentence may use the word imply where it should use infer. Below is a brief list of commonly confused words. (Many more are available in grammar textbooks and online.) You may want to review the definitions of these words in order to ensure that you are using them properly. accept (verb): to agree or take something offered except (preposition): excluding disinterested (adjective): impartial uninterested (adjective): not interested imply (verb): to indirectly suggest meaning infer (verb): to draw a conclusion disperse (verb): to scatter disburse (verb): to pay out than (conjunction): used for comparison then (adverb): next farther (adjective): more distant; more advanced further (adjective): in addition; extending beyond a certain point affect (verb): to produce an effect or change effect (noun): a result who (pronoun): referring to a person or to people which (pronoun): referring to a singular or plural thing that (pronoun): referring to things or a group of people conscience (noun): sense of right and wrong conscious (adjective): awake; aware

18

Try this practice question: Their were too many players on the field, so the referee called a foul.

A. NO CHANGE B. Their was C. There were D. There was

The correct answer is C. In this sentence, we need there rather than their. Don’t be fooled by choice D; we can’t use the verb was with the subject players. Double Negatives • In math, two negatives make a positive. The same is true in writing. • If we use two negatives, they cancel each other out. For example, consider the sentence “I don’t have no marbles.” If I don’t have no marbles, then I must have some marbles! • When you want to make a negative statement, you should use only one negative word. • Common negative words include nothing, none, never, not, neither, scarcely, barely, hardly, and without. Try this question: I couldn’t never have finished that difficult job by myself.

A. NO CHANGE B. could never have C. couldn’t hardly of D. couldn’t ever of

The correct answer is B. Since the sentence already utilizes a negative word (couldn’t), you don’t need the second one (never). Don’t be fooled by choice D. Although couldn’t ever would work in this sentence, we must say have finished rather than of finished. Idioms • An idiom is a word or expression that cannot be taken literally. For example, “He pulled the wool over my eyes.” • There are no rules to learn about idioms. They either sound right, or they don’t. • The most common idiom errors you will encounter are preposition errors. For example, it would be incorrect to say, “He pulled the wool across my eyes.”

19

Try this question: If you keep practicing the oboe, you’ll eventually get the hang for it.

A. NO CHANGE B. get a hang for it C. get a hang of it D. get the hang of it

The correct answer is D. The correct way to write the idiom is “the hang of it.” Paragraphs Some of the multiple-choice questions you’ll see on the English section of the ACT will deal with a paragraph as a whole. In those instances, you’ll be asked to identify sentences that are out of order, sentences that should be rewritten or combined with other sentences, and sentences that should be removed from the paragraph altogether. You’ll also be asked to select appropriate introductory and concluding sentences, transitional words, and supporting examples. Try this sample question: We visited a local animal shelter on Saturday. We wanted to pick out a new dog. As we walked through the corridor, each dog had its own way of sharing something about its personality. Some dogs barked, others jumped up and down, and still others cowered in the corners of their cages. We visited with several different dogs, but the one who stole our hearts was Bogart. This big, black dog with droopy ears and a wagging tail is now the newest member of our family. What is the best way to combine the first two sentences of this paragraph? A. We visited a local animal shelter on Saturday, we wanted to pick out a new dog. B. We visited a local animal shelter on Saturday because we were hoping we would be able to pick out a new dog. C. Visiting a local animal shelter on Saturday, we were wanting to pick out a new dog. D. We visited a local animal shelter on Saturday to pick out a new dog. The correct answer is D because it combines the two sentences without being wordy. Choice A, although not wordy, is a comma splice.

The Reading Section Most students will tell you that the reading section is the most difficult part of the ACT. Why? Because the lengthy passages require you to stay focused! In order to remain alert and to concentrate during the critical reading section of the ACT, you must remember Rule # 1: Stay engaged with the text. In other words, interact with the passages in front of you.

20

Here’s how: • First, write on the passage. Underline information that you think may be important. Underline transitional or signal words such as however, therefore, since, nevertheless, and above all. Circle words that are unfamiliar to you. Put stars next to examples the author provides. Make brief notes in the margins about the author’s purpose, point, or attitude. Don’t worry about marking details like dates and percentages, though, because questions that ask about specific details will refer you to the right line number anyway. Writing on the passage serves three important purposes. It helps you to make more sense of what you’re reading; it helps you to remember what you’ve read; and (here’s the clincher) it helps you to stay tuned in while you’re reading! • The second way to stay engaged with the text is to keep a conversation going in your head while you’re reading. Go ahead. No one else will know. Talk back to the author of the passage. Ask him questions like “What point are you trying to make?” or “Why did you describe the situation that way?” Make accusations like “Wow, you obviously don’t like this character very much” or “Well, I can tell you think global warming is nothing but a scam.” Get inside his head by saying “You’re trying to be sarcastic, aren’t you?” or “Oh, I see where you’re going with this example.” These conversations may feel awkward at first, but good readers have them all the time. They help you to think like the author (which means you’ll have an easier time answering the questions), and they help you to (once again) stay tuned in to the reading! Although interacting with the passages is important, don’t get too carried away. Keep in mind that you have a time limit, and try to follow these guidelines: • • • •

Go through the passage one time, marking it and talking to it as you go. Then go to the questions; Read through each one, and answer the ones you know; Go back to the ones about which you are unsure; When you go back to the passage to figure these out, remember that, generally, the questions follow the order of the passage. In other words, you should be able to find the answer to the first question near the beginning of the passage; • The last group of questions usually pertains to the passage as a whole.

You’ll have to answer questions about the point of the passage and how the author uses his words to make a point. Specifically, you need to practice figuring out a passage’s main idea, the author’s attitude or tone toward the subject matter, and what the passage implies (says indirectly or between the lines). Sometimes you will be asked to figure out the meaning of a word in context. In this case, the question will give you the line location of the word. Perhaps you will have already identified the word as one that is unfamiliar to you. Either way, you’ll need to read the sentence (and possibly the ones immediately before and after it) and look for context clues—words in the sentence that give away the meaning of the target word. You should also review your knowledge of root word, prefixes, and suffixes. Below are charts that will help you to do this. While you are not expected to memorize the information on the charts, familiarizing yourself with them will help you in understanding the difficult words you may encounter in the Reading Section of the ACT. 21

A Root is Worth a Thousand Words

© DPG Publishing, Inc. (Permission granted to copy this page for individual classroom use only.)

Important Prefixes and Suffixes

ROOT MEANING acerb bitter ag, act drive, urge, act agon contest, struggle alt high amic friend anim, anima 1. life 2. mind, soul, spirit arch rule, govern aug, auct increase autrui other people ban to command bel, bell 1. fair, fine 2. war bene well brev short cad, cas fall cand glow, burn cap, cip, cep to take ced, cess go, yield cens give opinion, appraise chron time ciarl prattle, babble cit, citat arouse, summon, call out clam, claim call out clement lenient, mild concili bring together copi abundance cor, cord heart corp body corrig to correct cred believe, make trust crit a judge cur care, attention curr, curs run cycl circle, wheel deleter destroyer derm skin didact to teach dol grieve, worry don gift, give dyn power enigm riddle equ 1. equal 2. horse erro, erron wanderer 22

ROOT MEANING exter, extra beyond, outside fac, fact, fec make, form, act, do fall, fals err, beguile, deceive fam 1. hunger 2. report fastidi loathing ferv boil firm steadfast, firm, strong flagr burn flex, flect bend fund, fus pour, melt garr chatter gen, gener origin, race, species geo earth ger bear, carry, rule graph, gram to write grav heavy greg flock, herd gyro turn helio sun hemera day hibit have, hold host, hosp host, guest humil humble hydr water inan void, empty it, itiner to go, journey judic judge jur 1. swear 2. law, right labor to work langu be weak leg, legis 1. appoint, send 2. law leth oblivion lev 1. light 2. lift, raise, rise line line listen desire liter letter log speech, word, reason loqu, locut speak, talk luc 1. shine 2. light lud, lus sport, play, laugh, mock luna moon magn great mal bad, ill malle hammer man hand mar, mer sea 23

ROOT MEANING mater, matr maternal metr, meter measure metus fear minn, minut diminish, lessen miser miserable mitt, miss to send, let go mor, mos will mord, mors bite morph form, shape mut change nat 1. born 2. swim nav, naus ship noc hurt, harm nounce bring tidings, tell nov new omin omen oper to work ox poison pac peace pais peace par 1. equal 2. get ready, set pars, part, partit divide, share, separate, part part father pat, pass suffer, feel, endure path feel, suffer pauc little ped 1. boy, child 2. foot pell, puls 1. drive, urge 2.skin, fur pet, petit to seek phan to show phem speech phone voice, sound pi 1. appease 2. devout plac please plaud, plaus clap hands ple, plet fill plic fold, bend, embrace potent powerful povre poor prehend, prehens take, seize, grasp prob, prov test prodigi token, omen prol offspring, increase pud feel shame, blush put 1. think 2. cleanse, lop off quiesc rest, become quiet 24

ROOT MEANING quir, quer to ask, seek rat think, calculate, settle rect ruled, right, straight rept creep riv stream rog, rogat ask, demand rupt to break sanct holy scind to cut scrib, script write scrut search into carefully sent, sens perceive, feel, think sequi to follow serv serve, keep solemn religious solv, solut loosen son sound spec, spect look, see, appear spond, spons promise, answer stat stand, standing stru, struct to build suav sweet sume, sump take, use, waste surrect, surreg rise syc fig tac be silent tedi irksomeness, irritating ten, tend, tent hold, stretch, strain termin boundary, end the 1. a god 2. place, put trepid trembling, agitated tric hindrances, wiles, snares trit rub trud, trus thrust turb disturb, drive un one ven, vent to come ver 1. spring 2. true, truth verb word vert, vers turn vi way, road vir 1. man 2. poison viv life, to live voc 1. voice 2. call vol 1. wish, will 2. fly

25

Important Prefixes and Suffixes PREFIX SUFFIX MEANING

a ab, abs ad, ac, af, ag, al, am, an, ap, ar, as, at ana ante anti, contra, counter apo arch, archi auto, aut be bi bio cata con, com, col, co, cor de deca, dec, deka, dek di dia dis epi eu ex, e, ec, ef, exo for hyper in, ir, el, em, en, il, im inter mal meta mis mono multi ob, oc, of, op octa over pan, omni par, per para peri poly

without, not; to, toward, into, at; from; out; off; on from, away to, toward, before, near up, back, again before against from, off chief self, same, automatic to cause two, both, double life down, thoroughly together, with down, from ten double through, between, across apart, away, un upon, to, besides good out, out of intensely, utterly over in, into between bad among, with, after, over wrongly, ill single, sole many to, toward, before eight above; superior; excessively all through beside around, about many

26

port, pros post preter pro, pur, pre proto quadri, quadr, quadra, quadru re, red, ana se se, sed, de, des, dis semi sub, suc, suf, sum, sup, sur, sus super, supra syl, sym, syn tra, trans tri un un, non, ir, in, il, im uni able, ible al ary, ory cle, cule ed er er, or est fy, ate ing ist ive ly or, ant, ar, ist, an, ian, ent ous phobia s, es sion, tion, ancy, ment, ency, ty, ance, ence, ity y, ey

27

toward after, behind beyond before, forward first four, square back, again aside, apart away, apart half under, after above, over, beyond together beyond, across three reverse not one able, capable (makes words into adjectives) referring (makes words into adjectives) place where (turns words into nouns) small makes a regular verb past tense makes the comparative degree one who performs a specific action makes the superlative degree to make (turns words into verbs) makes present participle form of verbs one who, that which inclined to (makes words into adjectives) like, resembling one who (turns words into nouns) full of (makes words into adjectives) excessive fear makes a word plural the state of, the act of (makes words into nouns) like, full of (makes words into adjectives)

Practice Questions Try interacting with this short reading passage and answering the questions that follow it. This passage is excerpted from a 1992 issue of National Geographic magazine. The United States and Canadian governments are mounting a defense against a Russian invasion. No, the clock has not been turned back. These invaders are Asian gypsy moths, foliagechomping insects that can cause billions of dollars in damage. Experts believe they entered North America from Russia last year in egg masses attached to grain vessels. The larger Asian gypsy moth is a more voracious feeder than the common North American strain and can feed on Pacific Northwest tree species. Unlike the flightless North American female, an Asian female can fly 20 miles between mating and egg-laying. The United States Health Inspection Service is barring from West Coast ports ships found carrying egg masses. The Tacoma and Portland areas, as well as Vancouver, British Columbia, have been sprayed with a biopesticide. 1. The author of this passage refers to the increase in Asian gypsy moths as a “Russian invasion” in order to

A. make a political statement. B. stress the severity of the problem. C. warn readers about underhanded activities of the Russian government. D. make light of a serious situation.

The correct answer to this question is B. By comparing the moth problem with a well-known historical situation, the author emphasizes the severity of the increase in the Asian gypsy moth population. Let’s take a look at the other choices. Choice A might trick some test-takers because the comparison is political in nature. However, the rest of the passage doesn’t deal with political matters at all. Choice C is incorrect for the same reason. Given the information the author provides in the passage, we can be certain that he isn’t making light of the situation (D). 2. Based on its use in the passage, the word voracious most likely means

A. speedy. B. able to go days without eating. C. picky. D. consuming large amounts of food.

The correct answer to this question is D. The Asian gypsy moth is a threat because it eats so much. The word larger is a good context clue because it suggests that these moths eat more than do common North American moths.

28

3. Which group of words from the passage best reflects the writer’s tone?

A. mounting, barring, sprayed B. egg masses, mating, egg-laying C. invaders, foliage-chomping, voracious D. Canadian, Russian, Asian, North American

The correct answer is C. We’re looking for subjective words here--words that the author chooses to express his attitude. Because the writer uses words like invaders, foliage-chomping, and voracious, we can tell that he is clearly concerned about this problem. Choices B and D offer purely objective words. The words mounting and barring in choice A seem tempting, but sprayed doesn’t fit the bill. ---------------------------------------------------------------------Here are some additional practice questions on which to test your skills. (Answers are on page 34.) I. This passage is excerpted from a 1986 article about the use of social studies textbooks in elementary schools. The inherent difficulty of social studies content stems mainly from the heavy technical concept load of social studies textbook passages. Technical concepts are one- or two-word “ideas” which have specialized meaning in social studies (for example: government, delta, immigrants, interdependence, economy, constitution, federal, cotton belt, division of labor, and political party). These words may have little or no meaning for students unless specific vocabulary or concept development lessons precede their first encounter with such terms. Yet basal social studies textbooks are notorious for heavy technical concept load and “thin” discussion of topics, making even the most careful independent reading low in potential benefit. Hard-to-pronounce names of cities, faraway countries, and foreign language names contribute to the complexity of textbook content. Many adult readers are stopped by these words, yet social studies is neither complete nor accurate without them. Add to the above problems frequent references to long periods of time or huge distances, and it becomes even more apparent why children have trouble learning from their social studies textbooks. What must a child of 9 or 10 think when the book says, “Our country was founded over 200 years ago”--or perhaps worse, “long, long ago”? What do expressions such as “far to the north” or “over a thousand miles to the east” mean to students who are not sure which direction is which and who have never traveled further than across the state or out of town? 1. The word “inherent” (line 1) means

A. naturally occurring. B. worst. C. least important. D. intentional.

29

2. According to the author of this passage, which of the following does not contribute to the ineffectiveness of social studies textbooks?

A. difficult terminology B. references to long periods of time C. uninteresting topics D. skimpy explanations of topics

3. The author’s main argument is that

A. social studies texts have no place in the classroom. B. social studies texts should be rewritten so that they are easier to understand. C. children today are not as intelligent as children in the past. D. the difficulties of social studies texts are necessary hurdles that must be overcome.

4. In the last paragraph of the passage, the author drives his point home with

A. rhetorical questions. B. imagery. C. hyperboles. D. an oxymoron.

II. This passage is excerpted from a 2003 article about the impact of electronic communication on writing. Word processing and e-publishing have brought about interesting developments in the way writers write. In general, the malleable nature of electronic text has made the physical process of composing more “elastic” in that writers are quicker to commit thought to writing and to reorganize content because it is simple to make changes on the electronic screen. Even young children find it easy to insert and manipulate images and video or audio clips in their texts. In addition, writers who publish on the Web perceive it as a new rhetorical space that provides options for using non-linear, alternative structures, making it necessary for them to anticipate how audiences might physically navigate through their hypertext compositions. This consciousness creates complex perspectives and a heightened awareness of traditional rhetorical elements in a way that text alone never could. 5. The word “malleable” (line 2) means

A. inflexible. B. simplistic. C. pliable. D. insincere.

30

6. The author of this passage suggests that word processing and e-publishing have made the writing and reading of text

A. more complex than ever before. B. quicker than ever before. C. more simple than ever before. D. more decorative than ever before.

III. This passage is excerpted from a 2002 article about ecosystems. When someone asks us where we are from or what we do, most of us mention the town or city where we live, our occupation, where we attended school, or our family heritage. We respond in terms of human communities, cultures, and geopolitical boundaries. We seldom, if ever, describe ourselves in terms of our ecological status in the natural world. We humans have so completely ordered, designed, and defined our physical environs and social milieu that our ecological connections have slipped from consciousness. Perhaps this is why we seem so unaware of our impact on nature and our rapid destruction of natural systems. We simply do not perceive ourselves as being part of the natural order of beings. All of us live within ecological systems, or “ecosystems,” and through our commerce, food distribution, and use of natural resources we each indirectly participate in the custodianship of many ecosystems worldwide. Ironically, we are simultaneously the most potent forces within most ecosystems, and yet nearly oblivious to the ecological effects of our daily lifestyles. There has never been a time when a deep understanding of ecosystems and our roles within them has been more critical. Indeed, the world’s freshwater ecosystems are so degraded that their ability to support plant and animal life, including humans, is viewed by many as being in peril. Learning about ecosystems is more than an expected focus in biology classes; it has become a study in survival. Ecosystems are functional units of interacting abiotic, biotic, and cultural (anthropogenic) components. All natural ecosystems are open systems where energy and matter are transferred in and out through the complex interactions of energy, water, carbon, oxygen, nitrogen, phosphorus, sulfur, and other cycles. Unfortunately, many scientists contend, we humans have disrupted the balance of transfers across ecosystem boundaries. In addition to learning our place within ecosystems, we must learn to become better stewards and managers of ecosystems. 7. The author’s tone in this passage can best be described as

A. apologetic. B. scolding. C. warning. D. encouraging.

31

8. The first two sentences of this passage are primarily intended to

A. capture the reader’s attention with an anecdote. B. demonstrate our unawareness of our place in the natural world. C. cause the reader to think about his or her own role in society. D. emphasize the importance of a person’s background.

9. The author’s attitude in this passage is represented by all of the following phrases except

A. “must learn to become better stewards and managers of ecosystems.” B. “oblivious to the ecological effects of our daily lifestyles.” C. “rapid destruction of natural systems.” D. “indirectly participate in the custodianship of many ecosystems worldwide.”

IV. This passage is excerpted from a 1922 novel about a Midwestern American’s journey to the front during World War I. Claude backed the little Ford car out of its shed, ran it up to the horse-tank, and began to throw water on the mud-crusted wheels and windshield. While he was at work the two hired men, Dan and Jerry, came shambling down the hill to feed the stock. Jerry was grumbling and swearing about something, but Claude wrung out his wet rags and, beyond a nod, paid no attention to them. Somehow his father always managed to have the roughest and dirtiest hired men in the country working for him. Claude had a grievance against Jerry just now, because of his treatment of one of the horses. Molly was a faithful old mare, the mother of many colts; Claude and his younger brother had learned to ride on her. This man Jerry, taking her out to work one morning, let her step on a board with a nail sticking up in it. He pulled the nail out of her foot, said nothing to anybody, and drove her to the cultivator all day. Now she had been standing in her stall for weeks, patiently suffering, her body wretchedly thin, and her leg swollen until it looked like an elephant’s. She would have to stand there, the veterinary said, until her hoof came off and she grew a new one, and she would always be stiff. Jerry had not been discharged, and he exhibited the poor animal as if she were a credit to him. Mahailey came out on the hilltop and rang the breakfast bell. After the hired men went up to the house, Claude slipped into the barn to see that Molly had got her share of oats. She was eating quietly, her head hanging, and her scaly, dead-looking foot lifted just a little from the ground. When he stroked her neck and talked to her she stopped grinding and gazed at him mournfully. She knew him, and wrinkled her nose and drew her upper lip back from her worn teeth, to show that she liked being petted. She let him touch her foot and examine her leg. When Claude reached the kitchen, his mother was sitting at one end of the breakfast table, pouring weak coffee, his brother and Dan and Jerry were in their chairs, and Mahailey was

32

baking griddle cakes at the stove. A moment later Mr. Wheeler came down the enclosed stairway and walked the length of the table to his own place. He was a very large man, taller and broader than any of his neighbours. He seldom wore a coat in summer, and his rumpled shirt bulged out carelessly over the belt of his trousers. His florid face was clean shaven, likely to be a trifle tobacco-stained about the mouth, and it was conspicuous both for good-nature and coarse humour, and for an imperturbable physical composure. Nobody in the county had ever seen Nat Wheeler flustered about anything, and nobody had ever heard him speak with complete seriousness. He kept up his easy-going, jocular affability even with his own family. As soon as he was seated, Mr. Wheeler reached for the two-pint sugar bowl and began to pour sugar into his coffee. Ralph asked him if he were going to the circus. Mr. Wheeler winked. “I shouldn’t wonder if I happened in town sometime before the elephants get away.” He spoke very deliberately, with a State-of-Maine drawl, and his voice was smooth and agreeable. “You boys better start in early, though. You can take the wagon and the mules, and load in the cowhides. The butcher has agreed to take them.” Claude put down his knife. “Can’t we have the car? I’ve washed it on purpose.” “And what about Dan and Jerry? They want to see the circus just as much as you do, and I want the hides should go in; they’re bringing a good price now. I don’t mind about your washing the car; mud preserves the paint, they say, but it’ll be all right this time, Claude.” 10. Claude’s attitude toward Jerry is one of

A. respect. B. resentment. C. jealousy. D. camaraderie.

11. The expression “imperturbable physical composure” suggests that Nat Wheeler

A. has a hard time keeping his face clean. B. is terribly overweight. C. cannot be irritated. D. always appears calm by his facial expressions.

12. Which statement by Mr. Wheeler represents an example of verbal irony? A. “You can take the wagon and the mules, and load in the cowhides.” B. “I shouldn’t wonder if I happened in town sometime before the elephants get away.” C. “They want to see the circus just as much as you do, and I want the hides should go in; they’re bringing a good price now.” D. “I don’t mind about your washing the car; mud preserves the paint, they say, but it’ll be all right this time, Claude.”

33

Answers to practice questions (Reading) 1. A. Because of the highly technical language of social studies, the difficult terminology discussed in this paragraph is a naturally occurring problem. 2. C. The passage specifically mentions all of the other problems, but at no time does the author suggest that social studies is not interesting. 3. D. The author states that the difficulties of the texts are inherent (or natural) and that social studies texts would be “neither complete nor accurate without” difficult words. Rather than passing judgment on the texts or on their readers, the author identifies necessary hurdles that must be overcome. 4. A. The author uses two rhetorical questions, or questions meant to make a point rather than to be answered. 5. C. The word “elastic” provides a good context clue, as does the statement that “it is simple to make changes on the electronic screen.” The word “malleable” suggests that the words on the screen are pliable, or can be easily altered. 6. A. This question is a little tricky because the author utilizes many of the words that are in the wrong answer choices (such as quicker and simple). None of these terms, however, represents his point. The use of alternative structures, images, and other nontraditional rhetorical elements makes electronic communication more complex than traditional texts. 7. C. The author is certainly not apologetic or encouraging. Although some of his words seem scolding, his overall purpose is more to warn than to scold. Therefore, warning is a better choice than scolding. 8. B. Although the opening sentences of the passage may do any of the things listed in the answer choices, their intended purpose is to show that we are oblivious to our place in nature. The focus of the rest of the first paragraph reinforces this purpose. 9. D. All of the other answer choices contain negative words and phrases that warn of threats to ecosystems. 10. B. Claude resents Jerry because Jerry injured a horse that is special to Claude. Evidence of Claude’s resentment can be seen in his description of Jerry’s carelessness and of the horse’s resulting injuries. 11. D. The sentence following the expression sums up its meaning: no one had ever seen Mr. Wheeler flustered. Although C is also a tempting choice, the author doesn’t say that Mr. Wheeler can’t be irritated, just that he never looks irritated. 12. D. Verbal irony involves saying the opposite of what you really mean. Mr. Wheeler teases Claude by suggesting that he would rather have a dirty car. 34

The Writing Test As mentioned in Chapter One, taking the ACT Plus Writing is your choice. This form of the test is available on two test dates of the national test. Your decision to take the Writing Test will depend on which colleges you wish to attend. The best way to find out if it is required is to seek the advice of your counselor or consult the colleges about their particular test requirements. Though the writing test is not required by all of the colleges to which you may apply, most will accept it—required or not. Try the following questions to test your skill in identifying errors in a written passage (answers are on page 37): My friend Ansley and me volunteer at a local animal shelter every weekend. We arrive at 1 the shelter at 9:30 a.m. and start walking the dogs. Most of them are real excited about 2 getting out of their cages. It’s usually difficult to walk the puppies because they don’t have no idea what they’re suppose to do while they’re outside. They usually just want to 3 4 play and roll around in the grass. Which is fine because we want them to have fun. While 5 6 we walk the dogs, the cages are cleaned by another volunteer. Then we return the dogs to 7 their cages and start our other chores including doing laundry, washing dog dishes, and 8 we mop the floors. After all of the chores are finished, we enjoy some playtime with the 9 animals. Each of the dogs need socialization, that way they can all find good homes 10 11 someday. 1.

A. NO CHANGE B. my friend Ansley and I C. me and my friend Ansley D. myself and my friend Ansley

2.

A. NO CHANGE B. get real excited about C. are really excited about D. are really excited over

35

3.

A. NO CHANGE B. they got no idea C. they have no idea D. they don’t got any idea

4.

A. NO CHANGE B. supposed to do while C. suppose to be doing while D. supposed to do any time that

5.

A. NO CHANGE B. grass. This is fine C. grass and that is fine D. grass, which is fine

6.

A. NO CHANGE B. another volunteer is who cleans the cages C. another volunteer cleans the cages D. the cages are being cleaned by another volunteer

7.

A. NO CHANGE B. doing laundry, and we wash dog dishes and also mop the floors C. doing laundry, washing dog dishes, and mopping the floors D. laundry, dog dishes, and mopping the floors

8.

A. NO CHANGE B. all of the chores is finished C. all the chores had been finished D. the finishing of all of the chores

9.

A. NO CHANGE B. All of the dogs need C. Each of the dogs is in great need of D. Each of the dogs are in need of

36

10.

A. NO CHANGE B. socialization, this will help them C. socialization, which will help them D. socialization seeing as how that way they can

11. The organization of this paragraph can be best described as

A. chronological. B. spatial. C. order of importance. D. cause and effect.

12. Which of the following sentences would make an effective conclusion for this paragraph?

A. All animals deserve a chance to find good homes. B. In conclusion, everyone should volunteer at an animal shelter on weekends. C. We volunteer at this same shelter every weekend. D. We’re exhausted when we finally go home, but we feel good about helping these homeless animals.

37

Answers to practice questions (Writing) 1. B. friend and I make up the compound subject of the sentence. Therefore, we must use the nominative case pronoun (I) rather than the objective (me). 2. C. Real is an adjective. We need really (an adverb) in this sentence to modify excited (a participle). 3. C. Don’t have no is a double negative. Although choices B and D eliminate the double negative as well, they use the incorrect verb form (got). Got must be used with a helping verb (such as have). 4. B. Don’t leave off the d in phrases like supposed to and used to. Although choice D uses supposed, it is wordier than choice B and therefore not the better choice. 5. D. As it is, which begins a sentence fragment. All of the answer choices correct this problem, but the choices are not all equal. Choice B presents a pronoun reference error (What does this refer to?). Choice C also presents a pronoun reference error (What does that refer to?) and is missing a comma before the conjunction that joins two independent clauses. 6. C. The original wording is passive voice, as is choice D. Choices B and C are active voice, but C is more concise and therefore a better choice. 7. C. The original wording is not parallel; rather, it consists of two gerund phrases and one independent clause. Choice C is the only one that uses parallel structure (three gerund phrases). 8. A. This sentence is correct as it is. Choice, B and C use wrong subject/verb agreement and wrong verb tense. Choice D is unnecessarily wordy. 9. B. The original wording is incorrect because the singular pronoun each does not agree with the verb need. Choice B corrects the subject/verb agreement problem and corrects a pronoun/ antecedent agreement problem that would otherwise exist with the next part of the sentence (each/they). 10. C. The original wording contains a comma splice, as does choice B. Choice C corrects the problem, while choice D is terribly wordy. 11. A. The paragraph starts at the beginning of the volunteer experience and moves chronologically (in order of time) through to the end. 12. D. Choice A could logically follow the sentence that is currently last, but it doesn’t relate to the paragraph as a whole. Because the paragraph focuses on the experiences of the author and her friend, choice B would be ineffective. Choice C merely repeats the topic sentence. Choice D provides closure for the whole paragraph.

38

The Essay Teaching students to write an essay is not the purpose of this guide. Instead, we will devote the remainder of this chapter to providing you with ACT’s criteria for evaluating the essay you will complete on the writing test. Information about scoring the essay, combined with the skill review provided above, should go a long way in helping you do well on the writing section of the ACT. According to ACT.org, “…Your essay will be evaluated on the evidence it gives of your ability to do the following: - express judgments by taking a position on the issue in the writing prompt; - maintain a focus on the topic throughout the essay; - develop a position by using logical reasoning and by supporting your ideas; - organize ideas in a logical way; - use language clearly and effectively according to the rules of standard written English; Your essay will be scored holistically—that is, on the basis of the overall impression created by all the elements of the writing. Two trained readers will read your essay, each giving it a rating from 1 (low) to 6 (high)…” 1 You can find more detailed information on how the essay is evaluated at the ACT website—www.act. org-The ACT Test. (Click on “The Writing Test Option.”) 1

ACT Writing Test. Essay Scoring Guidelines, http://www.act.org-The ACT Test

39

40

Chapter Four: Math Skills Review So many students approach this section of the test with an unhealthy amount of fear or anxiety. Familiarizing yourself with the format of the ACT math questions and becoming comfortable working with the equations you will need to answer the questions will help you overcome this. Approaching this part of the test with a positive attitude and a plan of attack will help you achieve your desired score. The mathematical reasoning sections are designed to test your problem-solving abilities in topics that are generally covered in the math classes taken by most high school students. These topics include pre-algebra, algebra, and geometry. Although you will be tested on your knowledge of these subjects, it is your ability to apply what you have learned that will determine your success on the test. Many of the formulas that you will need are provided for you on the test. You are being tested on your ability to apply these formulas when needed to arrive at the correct answers. Therefore, memorizing the formulas and their corresponding equations is not enough. Your comfort in applying your knowledge of what they mean is vital to success in solving mathematical problems on the test. Thy Days Are Numbered: Properties of Numbers A finite number of days remain before you are going to take the ACT, and it is always a good idea to begin preparing well in advance. Remember, you have been preparing for this day for several years now. This is where all of those hours spent in math classes finally pay off. For the majority of the mathematical reasoning questions, obtaining the correct answer will depend on your ability to manipulate numerical values. Approximately eighty percent (80%) of your answers will be a numerical value. In this section, you will review the types of numbers that may be encountered and some of their basic properties. Classes and Types of Numbers Yes, numbers do come in different types! We will begin by covering some of the basic terminology used to describe the different classes or types of numbers. • Counting Numbers The counting numbers, also referred to as the natural numbers, are probably the first numbers you ever encountered in life. Beginning with one, they continue infinitely in the positive direction. Examples: 1, 2, 3, 4, 5, …

41

• Integer The integers, also referred to as whole numbers, are the counting numbers together with their negatives (and zero). They continue infinitely in both the negative and positive directions. Examples: …, –4, –3, –2, –1, 0, 1, 2, 3, 4, … • Decimal The decimal system allows us to write numbers that are arbitrarily small. We can represent numbers or parts of a number that are less than 1 by using a decimal. The place values to the left of the decimal represent tenths, hundredths, thousandths, and so on. Examples: 1.2, 1.003, 0.00234, and 1.235 • Rational Numbers Rational numbers can be written as a ratio or fraction involving two integer. Any number that can be written as a terminating or a repeating decimal is a rational number! Examples: 1/2, 0.033, and 2 • Irrational Numbers Irrational numbers consist of any real numbers that are not rational numbers that is, they cannot be written as a ratio of two integers. Examples: √2, √3 , and π • Real Numbers The real numbers consist of all rational and irrational numbers. • Negative, Positive, or Not This is included here because you will often encounter this terminology when describing classes of numbers. o A negative number is any number that is less than zero. • -1, -√2, and -1/33 are negative numbers. o A positive number is any number greater than zero. • 2 , √3 , and 34/53 are positive numbers. o Zero is neither negative nor positive. o We cannot classify imaginary or complex numbers as positive or negative, although we can classify the real or the imaginary component. • 3-2i is neither positive nor negative, although the real component (3) is positive and the imaginary component (-2) is negative. o Non-negative numbers include zero and all positive numbers. • 0, 3, and √5 are examples. o Non-positive numbers include zero and all negative numbers. • 0, -π, and -1/28 are examples.

42

Multiples, Factors, and Primes A multiple of a number is any product of that number and an integer. So, 4, 8, 12, 16, and 20 are multiples of 4 because 4x1=4, 4x2=8, 4x3=12, 4x4=16, and 4x5=20. The test writers are typically going to consider the positive multiples of a number. They may even refer to the positive multiples as simply the multiples. However, multiples of a number can be negative as well. Note that -4, -8, -12, -16, and -20 are also multiples of 4. Multiples of a number need not be positive or negative. Although the chances it will come up on the test may be small, it certainly doesn’t hurt to know that 0 is a multiple of every number, since 0 is an integer and any number x0 = 0. The lowest (positive) multiple shared by two numbers is called the least common multiple. The multiples of 3 and 8 are

Multiples of 3: 3, 6, 9, 12, 15, 18, 21, 24, 27, 30, 33, … Multiples of 8: 8, 16, 24, 32, 40, 48, 56, 64, 72, 80, 88, …

As you can see, the first multiple to appear in both lists is 24. Therefore, 24 is the least common multiple of 3 and 8. A factor of a number is any (positive) integer that evenly divides (remainder of 0) into the number. The number has factors of 2, 3, 6, and 9 because 18/9 = 2, 18/6 = 3, 18/3 = 6, and 18/2 = 9. Any given number is a multiple of all its factors. The term greatest common factor is given to the largest factor shared by two numbers. The greatest common factor of 21 and 28 is 7, since 7 is the largest number that is a factor of both numbers. Although it is unlikely that you will simply be asked to factor a number for the test, being able to factor numbers will be important to answering many of the questions. A prime number is any integer greater than 1 whose only factors (positive divisors) are 1 and itself. Three (3) is a prime number because the only factors of 3 are 1 and 3. Six (6) is not a prime number because the factors of 6 are 1, 2, 3, and 6. Although we won’t get into why here, note that 1 is not considered a prime number. We can write every positive integer as a unique product of prime numbers. This is referred to as the prime factorization of a number. The number 4620 has a prime factorization of 2 • 2 • 3 • 5 • 7 • 11. The prime factorization of a number is unique; that is to say, there is no other combination of prime factors that will give the same product. This leads us to another concept that you may encounter, although probably not more than once on a given test. Two numbers are said to be relatively prime if they have no factors (other than 1) in common. The numbers 198 = 2 • 3 • 3 • 11 and 455 = 5 • 7 • 13 are relatively prime because they share no common factors in their prime factorization. When completely factoring a number begin by removing any obvious factors such as 2, 5, and 10. This will make it easier to examine any of the less obvious factors. In factoring a number, the following divisibility rules will be helpful. 1. One (1) is a factor of any whole number. 2. Any even number (ends in 0, 2, 4, 6, or 8) will be divisible by 2. • 312, 233, 247, 256 is divisible by 2, while 246, 468, 324, 221 is not. 43

3. Add up the digits in the number. If the sum of the digits is divisible by 3, then the number is divisible by 3 as well. Note that you can do this recursively for large numbers. • How would you determine if the number 937, 689, 865, 863 is divisible by 3? Add up the digits: 9 + 3 + 7 + 6 + 8 + 9 + 8 + 6 + 5 + 8 + 6 + 3 = 78. Is 78 divisible by 3? 7 + 8 = 15 and 15 ÷ 3 = 5, so the original number is divisible by 3. 4. If the last two digits are divisible by 4, then the number is divisible by 4. • 432, 235, 916 is divisible by 4 because 16 is divisible by 4. 5. Any number ending in 0 or 5 is divisible by 5. 6. Any even number that is divisible by 3 will be divisible by 6. (This is just the combination of rules 2 and 3 above) • 14 is divisible by 2 (because it ends in 4) but not by 3 (1 + 4 = 5, which is not divisible by 3), so 14 is not divisible by 6. • 15 is divisible by 3 ( 1 + 5 = 6, which is divisible by 3) but not by 2 (because it ends in 5), so 15 is not divisible by 6. • 18 is divisible by 2 (because it ends in 8) and by 3 (because 1 + 8 = 9, which is divisible by 3), so 18 is divisible by 6. 7. Double the last digit and subtract from the remaining digits. If this number is divisible by 7, then the original number is also divisible by 7. This can be done recursively as well. • To determine if 19,565 is divisible by 7, do the following. 1956 (5 x 2 = 10) – 10 194 (6 x 2 = 12) – 12 18 (2 x 2 = 4) – 4 14 / 7 = 2 8. If the last three digits are divisible by 8, then the entire number is divisible by 8. This 1 isn’t as helpful as some of the others. You can also just look for successive factors of 2 (remember that 2 • 2 • 2 = 8). • The number 257,392,104 is divisible by 8 because 104 is divisible by 8. Also, note that 257,392,104/2 = 128,696,052, 128,696,052 = 64,348,026, and 64,348,026/2 = 32,174.013. 9. This rule is similar to rule 3 above. If the sum of the digits is divisible by 9, then the number is divisible by 9. • 423,421,264,413 is divisible by 9 because the sum of the digits is 36 = (9 x 4). 10. Any number ending in 0 is divisible by 10.

44

Properties of Numbers Practice Problems 1. True or false: All natural numbers are non-negative. A) True B) False 2. Which of the following are positive integers? I. 2 II. 1/3 III. 0 IV. –3 V. π

A) I, II, III, and IV B) I, II, and III C) I and III D) I only E) II only

3. True or false: All non-negative integers are natural numbers. A) True B) False 4. Which of the following are non-positive rational numbers? I. -1 II. 0 III. –√2 IV. –√4 V. 1/3 A) I, II, III, and IV B) I, II, and IV C) I, II and V D) I, III, and IV E) I, and V

5. What is 1/500 written as a decimal? A) 0.01 B) 0.02 C) 0.05 D) 0.002 E) 0.005

45

6. Which of the following are multiples of 8? I. 24 II. 36 III. 4 IV. 1,624 V. 3,001

A) I, II, and III B) I, II, and IV C) I and IV D) II and IV E) I and V

7. The least common multiple of 10 and 18 is… A) 2. B) 80. C) 90. D) 180. E) 360. 8. Which of the following is not a factor of 20,790? A) 2 B) 3 C) 6 D) 11 E) 20 9. The greatest common factor of 874 and 1,995 is… A) 3. B) 7. C) 15. D) 19. E) 23. 10. The correct prime factorization of 22,770 is… A) 6 • 5 • 11 • 23. B) 2 • 3 • 5 • 11 • 23. C) 2 • 3 • 3 • 5 • 11 • 23. D) 2 • 3 • 3 • 3 • 5 • 7 • 11. E) 3 • 3 • 5 • 5 • 5 • 7 • 7.

46

11. Of the following pairs of numbers, which pair is mutually prime? A) 43,010 and 150,423 B) 3,705 and 7,116 C) 99,009 and 144,177,345 D) 68 and 13,247,352,132 E) 22 and 319,209,649 Note: Answers may be found on Page 109.

Frustrations with Fractions If fractions get you frustrated, you are not alone. Many students are uncomfortable working with fractions. A fraction is used to describe a part of a whole. The numerator (top) describes the part, and the denominator (bottom) describes the whole. In the fraction 2/3, the numerator is 2 and the denominator is 3. We’ll discuss operations such as addition and multiplication in a later section. For now, we just want you to familiarize yourself with the concepts of equivalent fractions, reducing fractions, and common denominators. Equivalent Fractions Stated simply, two fractions that describe the same part of the whole are considered equivalent fractions. If two friends cut a pie in half, they can each take 1/2 of the whole. What if they both cut their piece of the pie into two pieces? They now each have two pieces of the pie, which is now in 4 pieces. So, they each have 2/4 of the pie. 2/4 and 1/2 are equivalent fractions. In fact, 362/724 and 1/2 are equivalent fractions. Two equivalent fractions will be the same when completely reduced (written in simplest form). Both 2/4 and 362/724 simplify to 1/2. When one or both of the fractions contain large numbers it may not be in your best interest to try to reduce or simplify them, although you could certainly do this. We can compare two fractions by simply cross multiplying. For example, if we are asked to compare 1,728/4,032 and 3,972/9,268 we probably don’t want to simplify these fractions. You will want to cross multiply. (1,728 x 9,268 = 16,015,104) 1,728 3,972 (4,032 x 3,972 = 16,015,104) –––– x –––– 4,032 9,268 Because the cross products are the same, the fractions are equivalent. Had the product on the left been larger, the fraction to the left would be the larger of the two. Similarly, had the product on the right been larger, the fraction to the right would be the larger of the two. For example, 74/1001 1 > –3, 2 > √2

• Greater than or equal to: ≥ 8 /3 ≥ 2 2+3≥5 • Less than:
14. Then solve for x. The solution x > 7 is. In other words, “all values of x greater than 7” are solutions to this inequality. Example 2: Solve the inequality 3x + 5 < 5x – 9. First, subtract 5 from both sides to give 3x < 5x – 14. Then subtract 5x from both sides to give – 2x < – 14. To solve for x, divide both sides by – 2 and switch the inequality sign to give x > 7 as the solution. You may also be given graphical representations of inequalities.

65

Example 3: Which of the following algebraic inequalities is represented by the graph below?

-3 -2 A. B. C. D.

-1

0

1

2

3

–2≥y 5 Then solve for x in each equation. 2x < – 2 or 2x > 8 x < – 1 or x > 4

67

Thus, the solution to [2x − 3] > 5 consists of x < – 1 or x > 4. On a graph, this would be represented as the number line shown below, with open points at – 1 and 4 and arrows pointing in opposite directions to show x < – 1 and x > 4.

-2

-1

0

1

2

3

4

5

• Solving Rational Expressions A rational expression is the quotient of two polynomials (written as a fraction). For example: x–1 –––– x–2 You will likely be asked to solve equations or inequalities involving such expressions. Example: For what value of x is the following equation true? 4x ––––––– = 5 2x-1 First, multiply both sides by 2x – 1 to put the equation in linear form.

4x –––––– (2x – 1) = 5(2x – 1) 2x − 1 4x = 10x – 5

Solve for x.

– 6x = – 5 x = 5/6 Functions

A function is a rule or formula that tells how to relate elements in one set (the domain) with the elements in another set (the range). There are many different types of function notations that you should be familiar with. Here are a few examples: y = x2 f(x) = √x − 3 g(x)= 2x + 1/x

68

In each case, a specific value of x will give you a specific value of y, f(x), or g(x). Example 1: Given f(x) = x2 + 2x − 1, find f(2). When you see a question like this, all you need to do is substitute the new x value into the function. In this case, substitute 2 for x in the function.

f(2) = (2)2 + 2(2) − 1 =4+4–1 =7

2 Example 2: Given fx = 2x – 1 for x < 1 , find f(0) and f(1). x + 4 for x ≥ 1

This is called a “piecewise” function. The function is split into two halves: the half that comes before x = 1 and the half that goes from x = 1 to infinity. Which half of the function that you use depends on the value of x. If we evaluate f(0), we must use the first function, since 0 < 1. Then f(0) = 2(0)2– 1 = 0. If we evaluate f(1), we must use the second half of the function, since x ≥ 1. Therefore, f(1) = (1) + 4 = 5. Example 3: Given that f(x) = 3x2 + 2x, find f(x + h).

Everywhere there is an x, plug in x + h.

f(x) = 3x2 + 2x f(x + h) = 3(x + h)2 + 2(x + h) = 3(x2 + 2xh + h2) + 2x + 2h = 3x2 + 6xh + 3h2 + 2x + 2h Domain and Range • The domain of a function is the set of all values for which the function is defined. • The range of a function is the set of all values that are the output (result) of applying the function. _______ Example: What are the domain and range of f(x) = − √− 2x + 3 ? In this example, the domain is all the values that x can be. The only rule of this function is that you cannot take the root of a negative number. Therefore, by definition, – 2x + 3 ≥ 0.

69

First, to find all acceptable values of x, solve for x in – 2x + 3 ≥ 0. – 2x + 3 ≥ 0 – 2x ≥ –3 x ≤ 3/2 Then the domain is “all x ≤ 3/2.” To find the range in this example, plug in the highest value for x, which is 3/2. Thus, f(3/2) = 0. For any other value of x, f(x) will be negative. Therefore, the range is f(x) ≤ 0. Practice Problems for Manipulating Algebraic Expressions 1) If x = – 3, then 4x2 – 3x – 7 = ?

A) – 34 B) 20 C) 34 D) 38 E) 52

2) If 3x + 5y = 6 and 2x + y = 4, what is x?

A) 2 B) 0 C) – 1 D) 4 E) 5



A) 5 B) 2a C) 25 D) 5a E) 5a2

____ 3) Evaluate √25a2.

4) Factor 6x2 – 12x + 6 completely.

A) 6(x – 1)(x + 2) B) –6(–x + 4)(x + 2) C) 6(x – 1)(x – 1) D) 3(2x – 1)(x – 2) E) 2(3x + 2)(x + 1) 70

5) What is a possible solution of [2x − 3] – 4 = 3?

A) 2 B) – 2 C) – 5 D) 1 E) 3

6) Solve the following equation:

A) B) C) D) E)

3 4 5 6 7

x−1 2 ––––– = –– 15 5

Note: Answers may be found on Page 113.

How Things Change: Ratios, Proportions, and Variations This section deals with ratios, proportions, and how a quantity changes with regard to another quantity (variation). The topics build upon each other. Proportions are merely statements of equality involving ratios, and variation problems will often be written as proportions. Whether you are aware of it or not, you use ratios every day. When you talk about the rate of travel of a vehicle being 45 mph, you are talking about the ratio of distance traveled to time traveled. Ratios and Proportions Ratios are used to compare two quantities. They are typically written in one of two ways: as a fraction or with a colon (:). The ratio read as “4 to 10” could be written as 4/10 or 4:10. The ratio expressing the relationship between a day and a week would be written as 1/7 or 1:7 because there are seven days in a week. A proportion is a mathematical statement equating two ratios. Two ratios are said to be equal if, when written in fractional form, the fractions are equivalent fractions. We will compare and solve ratios using cross multiplication. If the cross products are equal, the ratios are equal. Take a look at the example below. x–1 x+2 –––– = ––––– 2 3

71

Cross multiply and solve for x. 3(x – 1) = 2(x + 2) 3x – 3 = 2x + 4



-2x

-2x



x–3=4



x=7



-3

+3

The correct answer is x = 7. We can check our answer by plugging it back into the original proportion. 7–1 7+2 –––– = ––––– 2 3 6 9 –– = –– 2 3



3=3

The final statement, 3=3, is a true statement. Therefore, x=7 is the correct answer. If you noticed that 6/2 and 9/3 are equivalent fractions, you could have stopped at this point. Two ratios are equal if their fractional forms are equivalent fractions. Remember, you may have to rewrite the proportion in fractional form before you solve it.

Solve for x : x: 6 = 4 : 12

Begin by rewriting the problem in fractional form; then solve for x as we did above.



x 4 –– = ––– 2 12 12x = 6 • 4

12x = 24



x=2



/12

/12

You probably could have solved this problem by observation. Recognizing that 4/12 is simply 1/3, we know that the left-hand side must be equal to 1/3 as well. One-third of 5 is 2. You may find that simple observations such as this will provide significant time savings on the exam. Variations Variations deal with explaining, in mathematical language, how one quantity changes with respect to one or more other quantities. The amount of flour needed in a bread recipe varies with the

72

number of loaves being made. The time it takes to paint a house varies with the number of people doing the work. The first is an example of direct variation; the second, of inverse variation. In the example of direct variation given above, the amount of flour needed for the recipe will vary directly with the number of loaves being made. Making more loaves will require more flour; making fewer loaves will require less flour. In direct variation, the variables will move in the same direction. As one variable increases or decreases, the other variable will move in the same direction. These problems can be solved using proportions. Consider the example below: If it takes 3 gallons of paint to cover 100 ft2, how many gallons of pint will be needed to cover 600 ft2? The problem gives us the ratio of 3 gallons of paint to every 100 ft√2. We can use this to set up a proportion problem. 3 x –– = ––– 100 600 100x = 3 • 600 100x = 1800



/100

/100

x = 18 It will take 18 gallons of paint to cover the 600 ft√2. The expression “y varies directly as x” could also br written:

y = kx

where k is called the constant of variation. In the previous problem, the number of gallons of paint varies directly with the square footage of wall that will be covered. The constant of variation is 3/100. We would then have 3 y = –––– x 100

3 = –––– 600 100 = 18

When two variables or quantities change in opposite directions, this is an example of inverse variation. In the example above about painting a house, the time required to paint the house varies inversely with the number of people painting. This means that the more people painting the house, the less total time it will take to paint. When we say “y varies inversely with x”, we can express this as k y = ––. x . 73

Again, k is the constant of variation. We can find k by simply multiplying the known values for the two variables. Consider the example below. A particular hotel has a custodial staff of 12 employees, and they can typically clean all of the hotel rooms in 6 hours. If four members of the custodial staff are not at work today, how long will it take the remaining custodians to clean all of the hotel rooms? In the example above, the total time taken to complete the job is inversely proportional to the number of workers. The constant of variation k is simply 6 • 12 =72. We can thus solve the problem as follows: 72 y = ––– x

72 = ––– 8 =9

The correct answer is 9 hours. I hope you recognized that the number of workers present is 8. There are four custodians absent, so x = 12 – 4 = 8. Be sure to read the information provided in the problems carefully. Two additional types of variation problems that we will discuss are joint variation and combined variation. These are just extensions of direct and inverse variation. When we say that “y varies jointly with x and z,” we have an example of joint variation. This can be written as

y = kxz ,

where k is again the constant of variation. The constant of variation is simply the ratio of y to x and z (x times z). Consider the problem below. The variable y varies jointly with x and z. The value of y is 12 when x is 4 and z is 8. What is y when x = 6 and z = 10? First, we want to be sure that we recognize the type of variation that is present. It is pretty simple for this problem: “y varies jointly with x and z.” We thus have the form:

y = kxy,

74

and we simply need to determine the value for the k. y k = ––– xy

12 = –––– 4•8 3 = –– 8

Now, we simply plug this into the equation of variation and solve for our unknown. 3 k = ––– xy 8

3 = –– • 6 • 10 8 45 = ––– 2 = 22 1/2

The correct answer is 22 1/2. Combined variation involves both direct and inverse variation. For example, if we say “y varies directly with x and inversely with z,” this is an example of combined variation. This can be written, similar to the joint variations previously presented, as x y = k ––– z Consider the following example. It takes 2 hours for 3 people to paint 100 ft. of 6-ft. fencing. Assuming that each person is capable of painting at the same rate, how long will it take for 12 people to paint 1,800 ft. of the same fencing? The total time (t) needed to paint a section of fencing is directly proportional to the length of the fence (l) and inversely proportional to the number of people (n) who are painting.

l t = k ––– n

75

We simply need to determine the constant of variation, k. tn k = ––– l

2•3 = ––– 100



6 = ––– 100

Now that we have k, we need to insert it into the equation of variation to get the answer. 6 l t = ––– • ––– l00 n

6 1800 = ––– • ––––– 100 12 = 9

The correct answer is 9 hours.

Ratios, Proportions, and Variation Practice Problems 1) Which of the following ratios represent the relationship of hours to days?

I. 24/1 IV. 1/24 II. 1:24 V. 24:1 III. 1 to 24



A) I, III, and V B) II, III, and IV C) II, III, and V

D) I and V E) II and IV

2) There are 14 boys and 16 girls in Tyler’s class. Which ratio best represents the relationship between the number of boys and the number of students in Tyler’s class?

A) 7/8 B) 7:10 C) 7/15

D) 14:16 E) 8:15

76

3) The ratio of f(x) to g(x) is 3:x + 2. If f(x) = x – 2, what is g(x) in terms of x?

A) 3x – 6 B) 1/3(x2 – 2) C) 2/3x

D) 3x2 – 12 E) 1/3(x2 – 4)

4) If the ratio of A to B is 3:4 and the ratio of B to C is 2:3, what is the value of A when C is 5? A) 10 D) 5 B) 3/2 E) 6 C) 5/2 5) If y varies directly with x and y = 4, what is the value of y when x = 7/11? A) 4/11 D) 44/49 3 E) 1 5/49 B) 2 /4 C) 11/18 6) If x varies inversely with y and y = 8 when x = 3, what is the value of x when y = 12?

A) 9/4 B) 1/2 C) 4 ½

D) 2 E) 4/9

7) The variable s varies jointly with t and u such that s = 7 when t = 4 and u = 3. What is the constant of variation describing the relationship between s on the one hand and t and u on the other? A) 84 D) 21/4 B) 12/7 E) 7/12 C) 4/21 8) The value of w varies directly with x and y and inversely with z. The constant of variation is 13/7. What is the value of z when w = 13, and y = 3?

A) 6/7 B) 7/13 C) 6/13

D) 7/6 E) 13/7

Note: Answers may be found on Page 109.

77

Playing the odds: Probability and Statistics

Some questions you are sure to encounter on the ACT math section will invoke your ability to calculate the likelihood (probability) of an event occurring and your ability to work with basic statistical measures (mean, median, and mode). We’ll review here a few of the most important concepts that you will need to be successful on these questions. Probability When answering these problems, keep in mind that the probability can be any number from 0 to 1 inclusively. If an event is not possible (cannot ever occur), the probability is 0. If an event is certain (will always occur), the probability is 1. You will never have a probability that is less than 0 or greater than 1. The probability formula can be used to calculate the probability of an event occurring. Number of times an event can occur Probability = ––––––––––––––––––––––––––––––– Total number of possible outcomes The probability is simply the ratio of the number of successful events divided by the total number of possible events. In a deck of 52 playing cards, the probability of pulling one of the 16 face cards is 15/52 or 4/13. In contrast, the probability of drawing one of the four jacks is 4/52 or 1/13. Dependent and Independent Events Calculating the probability of a single event is typically pretty simple. However, you may be asked to calculate the probability that two (or possibly more) events will both occur. In order to answer such a question, you will need to understand the difference between dependent and independent events. Events are considered dependent if the outcome of one event affects the probability of the other event. To calculate the probability of dependent events, we have to use something known as conditional probability. Conditional probability is the probability that an event will occur given that one or more other events have occurred. Take the generic example of three dependent events (A,B, and C). The probability that all three events will occur is given by the following expression P(A,B,C) = P(A) • P(B\A) • P(C\A,B). Don’t let this expression scare you. P(A) is simply the probability that event A will occur. P(B\A) is the probability that event B will occur given that event A has occurred. Likewise, P(C\A,B) is the probability that event C will occur if events A and B have occurred. The probability that all three events will occur, P(A,B,C), is simply the product of these three terms. This can be generalized to include more than three events. This will make more sense to you with an example. This weekend is the first round of the regional middle school soccer tournament. On Saturday morning, the team from Madris will play against the team from Dekalb. In the afternoon, Madris will play against the team from Fulton. On Sunday afternoon, Dekalb

78

will play against Fulton. In each game, either team has a 50 percent chance of winning the game. What is the probability that Madris will win both of its games and Fulton will lose both of its games? To solve this problem, we will have to use conditional probabilities. The probability that Madris will win its first game is 1/2. The probability that Madris will win its second game (against Fulton) is independent of the outcome of its first game. So, the conditional probability that Madris wins its second game given that it won it’s first game is still 1/2. The probability that Fulton will lose its first game is 1/2, but the conditional probability that Fulton will lose its first game given that Madris won both of their games is actually 1. Madris plays against Fulton in this game, so if we are given that Madris won this game then we know that Fulton lost this game. The probability that Fulton loses its second game is 1/2. Notice that this is also conditional probability. The likelihood that Fulton will lose its second game is independent of whether Madris won it’s games and independent of whether Fulton lost its first game. The probability that all four events occur (Madris wins its games and Fulton loses its games) is just the product of these probabilities. P(A,B,C,D) = P(A) • P(B/A) • P(C/A,B) • P(D/ A,B,C)

1 l 1 1 = ––– • ––– • 1 • ––– = ––– 2 2 2 8

The correct answer is 1/8. This also clarifies another concept, that of independent events. Take the first two events in the above problem for example. Madris, winning its first game and Madris, winning its second game are independent events. Events are considered independent if the outcome of one event has no effect on the likelihood of the other. The probability of two independent events is simply the product of the respective probabilities for these events. The probability that Madris will win both of its games is 1/2 • 1/2 , or 1/4. Statistics There won’t be too many problems that cover statistics on the test, but you can be sure there will be at least one. It is highly unlikely that any will cover topics beyond the three most basic statistical measures; mean (arithmetic average), median, and mode. The mean of a set of n numbers is just the sum of the numbers divided by n (the number of items in the set). To find the average of 15, 21, and 39, you would do the following: 15 + 21 + 39 75 –––––––––––– = ––– = 25 3 3 You may have to rely on your understanding of the properties of averages to arrive at an answer on the test when directly calculating the averages is difficult or time consuming. The average of two numbers is halfway between the two numbers on a number line. The average of 3 and 5 is 4. When you have a set of numbers evenly spaced, the average will simply be the middle number. The average of the first nine odd numbers will simply be the 5th odd number, or 9. When there

79

is an even number of terms in the set, the average is just the average of the two middle numbers. The average of the first 100 odd numbers is just the average of the 50th (99) and 51st (101) odd numbers. So, the average is 100. Two concepts that often appear on the test deal with weighted averages and finding a missing number when an average is given. The average of a set of numbers is just the sum of the numbers divided by the of terms number in the set. Sum of terms Average = –––––––––––– # of terms Likewise, we can use the average and the number of terms to calculate the sum of the terms.

Sum of terms = (Average) x (# of terms)

This principle is central to both calculating a missing term and to the concept of a weighted average. If we are given the value for three of the four terms (17, 35, 39) and the average (34) we can calculate the value of the missing term. Sum of terms = 17 + 35 + 39 + x = 34 x 4 = 136 91 + x = 136 x = 45 If we are given that (in a class of 16 girls and 13 boys) the boys averaged 75 on the last exam and the girls averaged 83, we can calculate the class average using a weighted average. Note that the average is not just the average of the boys’ and girls’ test averages.



Sum of Boy’s Scores + Sum of Girl’s Scores Class Average = –––––––––––––––––––––––––––––––––––––– total # of students



(Boy’s Average) x (# of Boys) + (Girl’s Average) x (# of Girls) = ––––––––––––––––––––––––––––––––––––––––––––––––––– # of Boys + # of Girls



75 x 13 + 83 x 16 975 + 1328 2303 = –––––––––––––– = –––––––––– = ––––– = 79.4 13 + 16 29 29

The average is slightly closer to 83 than it is to 75 because there are more girls in the class than boys.

80

Probability and Statistics Practice Problems 1) If a pair of fair dice is tossed, what is the probability that the sum of the two numbers is 4?

A) 1/6 B) 1/12 C) 1/2

D) 1/18 E) 1/9

2) If a pair of fair dice is tossed, what is the probability that the sum of the two numbers is an even number less than 10?

A) 1/3 B) 1/2 C) 4/11

D) 7/18 E) 2/3

Note: Answers may be found on Page 110.

Measuring Up: Geometry This section deals with the relationships between points, lines, and angles and the figures they form. The SAT has many questions involving the topics in this section. After algebra, this is the next most important section on the exam. Understanding the concepts will enable you to answer 30-40% of the questions that you will see on the exam. Definitions of Lines and Angles A line extends forever in either direction. The line below, called l, has three collinear points on it: A, B, and C. The part of the line between points A and B is called a line segment. In this instance, the line segment between points A and B will be referred to as either “segment AB” or simply AB. A and B are the endpoints of segment AB. A

B

C

l

Angles are most often described by the points on the lines that intersect to form the angle or by the point of intersection itself. For instance, in the diagram below, angle ABC could be described as either /_ABC or /_x. C x A

y B 81

A line forms an angle of 180°. If that line is cut by another line, it divides the 180° into two pieces that together add up to 180° (/_x + /_y = 180°). Two angles that when added together make 180°, are called supplementary angles (/_x and /_y are supplementary to each other). B A

C D

When two lines intersect, they form four angles, represented above by the letters A, B, C, and D. /_A and /_B together form a straight line, so they add up to 180°. The same is true for /_C and /_D, /_A and /_ D, and /_ B and /_ C. Since there are 180° above the line (/_A + /_ B), there are also 180° below the line ((/_C + /_D). Therefore, the sum of the four angles is 360°. /_A and /_C are opposite from each other and always equal to each other, as do /_ B and /_ D. These are known as vertical angles. In the figure below, /_x = /_y = 60°. x 120˚

120˚ y

When two lines meet so that 90° angles are formed, the lines are said to be perpendicular to each other. The 90° angle is called a right angle. A right angle is represented by a little box at the point of intersection of the two lines. Perpendicular lines are represented as l1 _| l2. 90˚

90˚

90˚

90˚

l1

l2 Two angles whose sum is 90°, or one right angle, are said to be complementary. For instance, in the figure below, /_AOB is the complement of /_BOC. Thus, /_AOB + /_BOC = 90°. A

B

C

O

82

Two lines in the same plane that are equally distant from one another at all points are called parallel lines. Parallel lines never meet. Parallel lines are often represented as l1 // l2. l1 l2 When parallel lines are cut by a third line (known as a transversal), eight angles are formed. A

D H

E

G

C

B

l1

F

l2

Based on what you’ve learned about parallel lines, /_A = /_C and /_B = /_D. Since the same transversal cuts line l2, the four angles /_E, /_F, /_G, and /_H are in the same proportions as the angles above. Thus, /_E = /_A and /_F = /_B. The next figure shows an example of the relationship of parallel lines cut by a transversal. 120˚ 60˚ 60˚ 120˚ 120˚ 60˚ 60˚ 120˚

l1 l2

Triangles A triangle is a three-sided figure whose angles always add up to 180˚. The largest angle of a triangle is opposite its longest side. On the exam, the triangle below will be represented as either “triangle abc” or ∆abc. c

50˚

a

60˚

70˚

83

b

On the exam, you will be expected to recognize specific types of triangles and understand the particular properties of each kind. Each kind is listed below with an explanation of the properties that you are expected to know. • Equilateral Triangles The three sides of an equilateral triangle are equal in length (a = b = c). The three angles are also equal, and therefore, since the three angles of a triangle must add up to 180˚, each angle of an equilateral triangle must equal 60˚ (x = y = z = 60˚). y a x

b z

c

• Isosceles Triangles An isosceles triangle is a triangle with two sides of equal length (a = b). The angles opposite the equal sides are also equal (x = y).

a

b y

x • Right Triangles

A if triangle is a triangle with a right angle. The other angles are, by definition, complementary angles (/_ Y + /_ Z = 90˚). The longest side of a right triangle (the one opposite the 90˚ angle) is called the hypotenuse (YZ or side c is the hypotenuse in the figure below). The other sides are often referred to as legs. y

a

b

x

b 84

z

Of all the triangles on the exam, right triangles appear the most often. Much information can be obtained from figures that contain right triangles due to the fact that their sides always exist in a particular proportion to each other. This proportion is better known as the Pythagorean theorem. (Note: Understanding this theorem is central to being successful on the exam.)

* The Pythagorean theorem is expressed in the following equation: a2 + b2 = c2,

where c2 is always the square of the hypotenuse (the side opposite the right angle) and a2 + b2 is the sum of the squares of the other sides. If you know the lengths of any two sides of a right triangle, you can use this equation to find the length of the last side. Remember: 1) the hypotenuse will always be the longest side, and 2) this works only for right triangles. In other words, if the triangle does not have a right angle, you cannot use the Pythagorean theorem. There are several “special” right triangles that frequently appear on the exam: 30˚-60˚-90˚ Triangles The lengths of a 30˚-60˚-90˚ triangle are in the ratio of 1:√3:2, as shown in the figure below. Short leg = x Long leg = x√3 Hypotenuse = 2x

B 30˚ x√3

C

2x

x

60˚

A

For example, if you know the length of the hypotenuse is 4 and the length of the short leg is 2, then the Pythagorean theorem gives you the length of the longer leg: c2 = a2 + b2 c = 4, a = 2 42 = 22 + b2 16 = 4 + b2 12 = b2 2√3 = b

85

45˚-45˚-90˚ Triangles A 45˚-45˚-90˚ triangle is an isosceles triangle containing a right angle. Thus, the two legs of the triangle (opposite the 45˚ angles) are equal. The sides of the triangle are in the ratio of 1:1:√2, as shown in the figure below. B 45˚ x

x√2 C

C

x

x 45˚

A

• Area of a Triangle The area of a triangle is defined as half the base (b) of the triangle times the height (h) of the triangle, which is represented by the formula below: area∆ = 1/2bh Height is the perpendicular distance from the base of the triangle to its highest point. In the first two triangles below, the height is represented by a dashed line. The height of the triangle can be outside the triangle itself, as in the second example below. In the right triangle, the base and height are just the legs of the triangle.

86

• Similar and Congruent Triangles Two triangles are similar if their angles have the same degree measures. The triangles therefore have the same shape, and their sides will be in proportion. For example, the two triangles below are similar. Both are 45˚- 45˚- 90˚ right triangles, therefore; each side of one triangle is in proportion to the corresponding side of the other triangle. Here, the first triangle is three times as large as the second triangle since the sides are in a ratio of 3:1.

6√2

6 6

2

45˚

45˚

2√2 2

Two triangles are similar if: 1) Two pairs of corresponding angles have the same measure. 2) One pair of corresponding angles has the same measure, and the pairs of corresponding sides that form those angles have lengths that are in the same ratio. Congruent triangles are triangles that have the same size and shape. x A B F z

y

z

y

D

C

x

E

Each side of ∆ABC has the same length as the corresponding side of ∆DEF. AB = DE = x BC = EF = y CA = FD = z Each angle of ∆ABC is equal to its corresponding angle in ∆DEF.

87

Here’s a quick checklist to determine if two triangles are congruent. 1) Each pair of corresponding sides has the same length. 2) Two pairs of corresponding sides each have the same length, and the angles formed by these sides have the same measure. 3) One pair of corresponding sides has the same length, and two pairs of corresponding angles each have the same measure. Note: If any of these three details is true, then you’re dealing with two congruent triangles.

Quadrilaterals and Other Polygons A quadrilateral is a four-sided polygon whose angle sum to 360°. There are several types of quadrilaterals that you will see on the exam. You will be expected to utilize the information that these figures offer in order to solve many types of questions. The perimeter of any polygon is the sum of the lengths of its sides. The most common four-sided figures are the rectangle and the square, followed by the parallelogram and the trapezoid. • Parallelograms A parallelogram is a four-sided figure in which both pairs of opposite sides are parallel, both pairs of opposite angles are equal, and both pairs of opposite sides are equal in length. 4 3

3 4

The area of a parallelogram is its base times its height, but due to its shape, the height of a parallelogram is not always equal to one of its sides. To find the height of a parallelogram, you must draw a perpendicular line from the base to the top of the figure. Area of a parallelogram = base x height

88

• Rectangles and Squares A rectangle is a parallelogram that has four interior angles that are each equal to 90˚. Opposite sides of a rectangle are equal. The diagonal of a rectangle makes two equal right triangles. The Pythagorean theorem can be used to figure out details regarding the length of sides or the diagonal depending on what information you’re given. The area of a rectangle is its length times its width. In the figure below, the area = 6 x 3, or 18. 6

3

3

6 Area of a rectangle = length x width A square is a type of rectangle whose four sides are equal in length. It is important to notice that the diagonal of a square makes two 45˚- 45˚- 90˚ triangles with the sides of the square. Thus, you can figure out the length of the sides from the length of the diagonal or the length of the diagonal from the length of a side. 3 3 Since a square is a rectangle, the area of a square is also length times width. However, since each side s is the same, the area of a square can be represented as: Area of a square = s2 • Trapezoids A trapezoid is a quadrilateral having one pair of parallel sides. Since AB // DC, x + y = 180° and w + z = 180°. A

D

w

x

B y

z

C

The easiest way to find the area of a trapezoid is to divide it into two triangles and a rectangle, figure out the areas of the individual pieces, and then add the results together to find the area of the whole figure.

89

Circles A circle represents all the points at a distance r from any given point. The measure of degrees around the point of A

C B

O

D

• Definitions

F

E

A chord is a line connecting two points on a circle (BD and EF are chords). The diameter of a circle is a line segment that passes through the center and has its endpoints on the circle, as in DOB. All diameters of the same circle have equal length. The radius of a circle is a line segment extending from the center of the circle to a point on the circle. All radii of the same circle have equal length (OA and OB are both radii). The radius is also half the diameter. An arc is a part of a circle. An arc can be measured in degrees or in units of length. (AB is an arc.) If you form an angle by drawing radii from the ends of the arc to the center of the circle, the number of degrees in the arc equals the number of degrees in the angle formed. A tangent to a circle is a line that touches the circle at only one point. A tangent is always perpendicular to the radius that contains the point of the line that touches the circle. AC is a tangent to circle O. The circumference is the measure of the distance around the circle. It is equal to π times the diameter d (or π times twice the radius r). Circumference = πd or Circumference = 2πr The area of a circle is equal to π times the square of the radius. Area of a circle = πr2

90

Graphing and Coordinate Geometry You will be expected to answer questions involving both linear and quadratic equations and their graphs. Therefore, you will need to understand the basics of the coordinate plane. • The Cartesian Grid Every real point (x,y) has a place on this grid. For instance, the point A (3,1) can be found by counting over on the x-axis three places to the right of the origin (0,0) and then counting on the y-axis one place up from the origin. Note: Always remember: moving up on a grid means x is becoming more positive, while moving left on a grid means y is becoming more positive. For instance, when moving from point C to point B, you must go from –2 to 2 on the x-axis and from –3 to –1 on the y-axis.

A

• (3,1)

B

• (2, –1)

C

• (–2, –3)

The Cartesian grid can be broken up into four quadrants with respect to the x-axis and y-axis. The signs of x and y change depending on the quadrant in which the point lies. The quadrants are labeled from I to IV in a counterclockwise direction. In Quadrant I (top right), both x and y are positive. In Quadrant II (top left), y is positive whereas x is negative. In Quadrant III (bottom left), the values of both x and y are negative. Finally, in Quadrant IV (bottom right), x is positive while y is negative. The importance of this concept will become more apparent later when reflecting points or lines are discussed, since the signs in Quadrants I and III are exact opposites (the same is true for Quadrants II and IV). II I x is negative y is positive

x is positive y is positive

x is negative y is negative

x is positive y is negative

III

IV

91

Finding the midpoint between two points You will be expected to find the midpoint of line segments in the coordinate plane. The midpoint (xm, ym) is the average of the x’s and average of the y’s: x1 + x2 y1 + y2 xm = –––––– ym = –––––– 2 2 For instance, if AB has endpoints A (4,5) and B (2,3), then the midpoint of AB has the coordinates 4 + 2 • ––––––) 5 + 3 = (3, 4) (–––––– 2 2 Finding the distance between two points The Pythagorean theorem can be used to find the distance between any two points in the coordinate plane. Take the figure below. B(4,4)

4 3 2 1 -4 -3 -2 -1

A(1,1)

0 -1 -2 -3 -4

1

C(4,1)

2 3 4

Points A, B, and C form a right triangle, ΔABC, where AC = 3 and CB = 3 (thus, ΔABC is an isosceles right triangle). Applying the Pythagorean theorem: AB2 = 32 + 32 AB2 = 18 AB = √18 = 3√2 Therefore, the distance between two points (x1, y1) and (x2, y2) in the coordinate plane can be summarized by the formula Distance between two points = √(x2 − x1)2 + (y2 − y1)2

92

• Equations of lines (y-intercept form) The equation of a line can be found by using the formula y = mx + b, where x and y are represented by the point (x, y), m is the slope of the line (how sharply a line is inclining or declining), and b is the y-intercept (the point where a line crosses the y-axis). The slope of a line between two points (x1,y1) and (x2,y2) is defined as follows: rise change in y = y1 − y2 Slope = ––––– = ––––––––––– = ––––––– run change in x = x1 − x2 Below is the graph of the line y = 2x + 1. The line crosses the y-axis at a point 1 above the origin (y = 1). The slope is 2. An easy way to think of the slope is as the fraction 2/1, where 2 represents the direction of movement on the y-axis and 1 represents the direction of movement on the x-axis. Thus, from y = 1 on the y-axis, you would move up 2 and right 1. y = 2x + 1

If the equation of a line is y = – 3/2x – 1, you would start by moving 1 down from the origin to y = –1. If you interpret the fraction as − 3/2 , then you would move 3 down and 2 to the right to give you the graph shown below.

y = – (3/2)x – 1

93

Key relationships A positive slope represents an inclining line (y values increase from left to right). A negative slope represents a declining line (y values decrease from left to right). Parallel lines have equal slopes. For instance, the lines y = 3x +1 and y = 3x – 2 are parallel lines because both lines have a slope of 3. Two lines are perpendicular when the product of their slopes equals –1. The lines y = 2x + 1 and y = –1/2x – 3 are perpendicular lines because the product of their slopes equals –1 (1 • (–1/2) = –1). • Graphs of Quadratic Functions You will be expected to be able to identify some of the features of the graph of a quadratic equation, such as its highest or lowest point, its solutions and its direction. The equation of a quadratic function is expressed as: y = ax2 + bx + c, where a, b, and c are all constants and a ≠ 0. The graph of a quadratic function is called a parabola. A parabola is a U-shaped curve that can open upward or downward depending on the sign of a. If a > 0, then the graph will open upward. If a < 0, then the graph will open downward.

a>0 a 0?

A) – 4/3 < x < 4/3 B) x < 4/3 or x > 4/3 C) x > 4/3 D) x > – 4/3 E) all real numbers

19) If s2 = 19, what is the value of (s + 1)(s – 1)?

A) √19 – 1 B) √19 + 1 C) 18 D) 20 E) 35

20)

The table above shows some of the values for the function f. If f is a linear function, what is the value of the x-intercept in terms of a, b, and c?

A) –2a B) a – c C) 2a – b D) b – c E) 0

153

21) Sarah has at least one quarter, one dime, one nickel, and one penny. If she has three times as many pennies as nickels, the same number of nickels as dimes, and twice as many dimes as quarters, what is the least amount of money she could have?

A) $0.41 B) $0.48 C) $0.58 D) $0.61 E) $0.71

22) In the figure, DA bisects _/ BAC, and DC bisects _/ BCA. If _/ ADC = 120°, what is the measure of _/ B?



Note: Figure is not drawn to scale.



A) 40 B) 60 C) 80 D) 100 E) 120

23) A triangle has base 9, and the other two sides are equal. If the side lengths are integers, what is the shortest possible side?

A) 1 B) 2 C) 3 D) 4 E) 5

154

24) A bag contains only white and red marbles. The probability of selecting a red marble is 1/4. The bag contains 200 marbles. If 50 white marbles are added to the bag, what is the probability of selecting a white marble?

A) 1/3 B) 4/5 C) 2/5 D) 2/3 E) 3/5

25) In the figure, line L is parallel to line M, with angles a, b, c, d, e, f, g, and h as shown. Which of the following lists includes all of the angles that are supplementary to _/ a?



A) b, d, and c B) b, d, f, and h C) c, e, and g D) d, c, h, and g E) e, f, g, and h

26) The set B consists of all even integers between –34 and 2m. If the sum of these integers is 74, what is the value of m?

A) 19 B) 36 C) 37 D) 38 E) 40

27) Which graph represents the solution set of [5 + 2y] ≥ 3?

155

28) A 12-ton mixture consists of 1/6 sand, 2/6 gravel, and 3/6 cement. If x tons of cement are added, the mixture will contain 60% cement. How many tons of cement need to be added?

A) 1.2 B) 3 C) 3.2 D) 4 E) 5.2

29) The points A, B, C, D, and E lie at (–4,0), (2,0), (0,4), and (0,–5), respectively. Which of the following line segments has the greatest length?

A) AD B) BD C) AE D) AC E) CE

30) In the figure, MO _| LN, LO = 2, MO = ON, and LM = 4. How long is MN?



A) 2√6 B) 2√3 C) 3√2 D) 3√3 E) 3√6

31) How many points of intersection exist if the equations (x – 5)2 + (y – 5)2 = 4 and y = –x are graphed on the same coordinate plane?

A) 0 B) 1 C) 2 D) 3 E) 4

156

32) Which of the following graphs best represents [y] = x + 1?

33) At what point (x,y) do the two lines with equations y = 3x – 1 and 10x – 3y = 9 intersect?

A) (6,17) B) (8,23) C) (12,35) D) (17,6) E) (35,12)

34) In the figure, what is the area of ∆ABC in terms of x?



A) 60 cos x B) 120 cos x C) 24 tan x D) 60 sin x E) 120 sin x

157

2

35) If 16x

−1

= 64x, what is x?

A) 2/3 B) − 2/3 C) 3/2 D) − 3/2 E) - 1/2

36) In the xy-plane, the equation of line l is y = –3(x + 2)2 + 4. If line m is the refection of line l in the y-axis, what is the equation of line m?

A) y = 3(x − 2)2 – 4 B) y = – 3(x − 2)2 + 4 C) y = 3(x + 2)2 + 4 D) y = 3(x + 2)2 – 4 E) y = 3(2 − x)2 + 4

37) What is the value of (x + y)2 if x2 + y2 = 10 and 4xy = 8?

A) 4 B) 10 C) 14 D) 18 E) 100

38)

If the area of the square in the figure above is 81 and the perimeter of each of the 4 triangles is 30, what is the perimeter of the figure outlined by the solid line?

A) 68 B) 74 C) 76 D) 81 E) 84

158

39) If rstv = 1 and stuv < 0, which of the following must be true?

A) r > 0 B) s < 1 C) t < 0 D) u ≠ 0 E) v ≠ 1

40) The center of a circle if M (0,2), and the endpoint of one of its radii is A (-6,-4). If AB is a diameter, what are the coordinates of B?

A) (8,6) B) (6,8) C) (4,6) D) (6,4) E) (8,10)

41) There are eight sections of seats in an auditorium. Each section contains at least 300 seats but not more than 400 seats. Which of the following could be the number of seats in the auditorium?

A) 1600 B) 2000 C) 2200 D) 2600 E) 3400

42)

In the figure above, each square is tangent to the containing circle at only one point. If the area of each square is x2, what is area of the shaded region in terms of x?

A) (π-2)x2 B) (π-4)x2 C) (4-π)x2 D) (π-1)x2 E) 2(π-2)x2

159

43) How many complete tanks of water, each with a capacity of 3 cubic meters, are needed to fill an empty cylindrical tank whose height is 3 meters and whose base has a radius of 2 meters?

A) 12 B) 13 C) 14 D) 15 E) 16

44) During a game, the green team scored one-eighth of its points in the first quarter, one-third in the second quarter, one-fourth in the third quarter, and the remaining points in the fourth quarter. If its total score for the game was 48, how many points did the green team score in the fourth quarter?

A) 18 B) 14 C) 12 D) 10 E) 7

45) At a meat processing plant, chickens are accepted only if they weigh between 3.65 and 4.35 pounds. If the plant accepts a chicken weighing m pounds, which of the following describes all possible values of m?

A) [m − 4] = 0.35 B) [m + 4] = 0.35 C) [m − 4]< 0.35 D) [m + 4] < 0.35 E) [m − 4] > 0.35

46) In the figure below, if AB = 10, what is the value of k?

B(8,k)



A) 6 B) 8 C) 10 D) 12 E) 18

A O

C

160

47) 3, 7, -7,… The first term in the sequence of numbers shown above is 3. Each even-numbered term is 4 more than the previous term, and each odd-numbered term after the first is –1 times the previous term. For example, the second term is 3 + 4, and the third term is (–1) x 7. What is the 155th term of the sequence?



A) –7 B) –3 C) 1 D) 3 E) 7

48) Dwayne has a woodshop from which he builds and sells birdhouses. He sells each birdhouse for k dollars. Out of this, one-third is used to pay for the lumber and supplies, and he saves the rest of the money. In terms of k, how many birdhouses must Dwayne sell to save $5000? A) 15000/k B) k/15000 C) k/7500 D) 7500/k E) 7500k 49) The sum of 12 consecutive integers is 5,250. What is the value of the greatest of these integers?

(A) 440 (B) 441 (C) 442 (D) 443 (E) 444

50) If L is parallel to M, what is the value of y?

(A) 25 (B) 50 (C) 130 (D) 155 (E) 180

130˚ x˚ x˚



L M

161

51) The table below gives values of the quadratic function h for selected values of x. Which of the following defines h? x h(x)

0 1

1 2

2 5

3 10

A) h(x) = x2 + 1 B) h(x) = x2 + 2 C) h(x) = 2x2 – 2 D) h(x) = 2x2 – 1 E) h(x) = 2x2 + 1

52) In the figure below with point A as its center, AB =10 and AC = 4√2 . What is the area of the shaded region? C



A) 100π + 32 B) 64π + 128 C) 36π + 128 D) 100π +128 E) 36π + 32

B

4√2 10 A

53) What is the surface area of the rectangular prism below?



A) 312 B) 336 C) 360 D) 384 E) 432

54) What is cos 0 if tan 0 = 4/3?

A) 3/5 B) 3/4 C) 4/5 D) 5/4 E) 5/3

162

55) A coin was flipped 24 times and came up heads 14 times and tails 10 times. If the first and the last flips were both heads, what is the greatest number of consecutive heads that could have occurred?

A) 7 B) 9 C) 10 D) 13 E) 14

56) If a rectangle’s length and width are both doubled, by what percent is the rectangle’s area increased? A) 50 B) 100 C) 200 D) 300 E) 400 57) If a small cube (side = 2 inches) is filled twice and poured into a larger cube (side = 4 inches), what will be the height, in inches, of the water in the larger cube?

A) B) C) D) E)

/4” 1 /2” 1” 2” 4” 1

58) Point Q lies at the center of the square base (ABCD) of the pyramid below. The pyramid’s height (PQ) measures exactly one-half the length of each edge of its base, and point E lies exactly halfway between C and D along one edge of the base. What is the ratio of the surface area of any of the pyramid’s four triangular sides to the surface area of the shaded triangle?



A) 3 : √2 B) √5 : 1 C) √3 : 3 D) 2√2 : 1 E) 8 : √5

163

sin20 59) Simplify ––––––––. 1 − cos20

A) sin 0 B) cos 0 C) tan 0 D) 0 E) 1

60) On the xy-coordinate plane, all of the following points lie on the circumference of a circle whose radius is 10 and whose center lies at (–1, 0) EXCEPT:

A) (–6, –7) B) (–2, 3√11) C) (–1, –10) D) (7, 6) E) (1, –4√6)

164

Science Test 40 Questions, 35 Minutes (Answers may be found on Page 212.) Directions: Each of the following seven passages is followed by several questions. After reading each passage, decide on the best answer to each question and fill in the corresponding oval on your answer sheet. You are allowed to refer to the passages while answering the questions. Calculator use is not allowed on this test. Passage I Solubility refers to the ability for a given substance, the solute, to dissolve in a solvent. It is measured in terms of the maximum amount of solute dissolved in a solvent at equilibrium. The resulting solution is called a saturated solution. Under various conditions, the equilibrium solubility can be exceeded to give a so-called supersaturated solution. The solubility of a given solute in a given solvent often depends on temperature. The chart below shows solubility curves for some typical inorganic salts.

1) How many grams of NH4Cl can be dissolved in 200 g of water at 70 ˚C?

A. 70 B. 90 C. 120 D. 180

2) As the temperature of a solution of NaNO3 is increased, what trend in its solubility is observed?

A. Decreasing solubility B. Increasing solubility C. No change in solubility D. Temperature does not affect its solubility.

165

3) At what temperature is the solubility of sodium chloride (NaCl) the same as the solubility of potassium chloride (KCl)?

A) 22 B) 30 C) 34 D) 40

4) What compound is least soluble at 40 ˚C?

A) B) C) D)

NaCl KCl Na2SO4 NH4Cl

5) Which procedure will increase the solubility of KCl in water?

A) stirring the solute and solvent mixture B) increasing the surface area of the solute C) raising the temperature of the solvent D) increasing the pressure on the surface of the solvent

Passage II Lasers emitting visible light are the most common; the basic principles are applicable through much of the electromagnetic spectrum. The first stimulated emission of light was achieved in the microwave region of the spectrum, but now lasers are available that emit light in the ultraviolet, infrared, and x-ray spectral regions. Lasers range in continuous power output from less than a milliwatt to many kilowatts, and some produce trillions of watts in extremely short pulses. Experiment 1 Research shows that a photon of light is absorbed by an atom in which one of the outer electrons is initially in a low-energy state. The energy of the atom is raised to the upper energy level and remains in this excited state for a period of time — typically less than 1 microsecond. The electron then drops to the lower state through the emission of a photon of light. The best energy output is achieved when, for every photon absorbed, another is emitted. Experiment 2 If an atom or molecule lies in an energy state that is higher than the lowest, or ground level, state, it can spontaneously drop to a lower level without any outside stimulation. One possible result of dropping to a reduced energy state is the release of the excess energy (equaling the difference in the two energy levels) as a photon of light. 166

Experiment 3 Electrons can be bumped up to higher energy levels by the injection of energy ­— for example, by a flash of light. When an electron drops from an outer to an inner level, “excess” energy is given off as light. The wavelength or color of the emitted light is precisely related to the amount of energy released. 6) Which of the following is not supported by the experimental results? A) The amount of energy released when an electron drops energy levels correlates with the wavelength of the light emitted. B) Lasers can emit only visible light. C) Lasers can emit light in the ultraviolet, infrared, and x-ray spectral regions. D) A photon of light can be absorbed by an atom. 7) Energy can be directly released from all of the following EXCEPT:

A) an electron in an excited state B) an outer electron in a higher energy state C) an electron in its lowest energy state D) a photon being emitted

8) Which experiment concerns atoms or molecules initially in a lowered energy state?

A) Experiments 1 and 2 B) Experiments 2 and 3 C) Experiments 1 and 3 D) None of the experiments

9) Which experiment(s) DO(ES) NOT require any energy to be added in order for energy to be given off?

A) Experiment 1 B) Experiment 2 C) Experiment 3 D) All of the experiments

10) What experiments offer the most promise for use in lasers (assuming the same atom or molecule)?

A) Experiments 2 and 3 B) Experiments 1 and 2 C) Experiments 1 and 3 D) Unable to determine based on information given.

167

11) What is the most telling way to determine if an atom or molecule has the best potential for use in lasers?

A) Measure the energy level of the initial atom or molecule. B) Measure the energy level of its excited state. C) Measure the amount of energy added to reach its excited state D) Measure the amount of energy given off after going from its excited state to a lower energy level.

Passage III The graphs below show annual mean global temperature anomalies over the period 1880-2001. The zero line represents the long-term mean temperature from 1880-2001, and the bars or lines show annual departures from that mean.

12) What overall trend, in terms of global temperature, is observed since the beginning of the 20th century?

A) decreasing temperature B) increasing temperature C) no change D) increasing, then decreasing temperature

168

13) Which of the following statements is true based on the data in the graph?

A) Ocean temperatures have greater anomalies than those observed on land. B) Ocean temperatures have the same amount of anomalies as those observed on land. C) Land temperatures have greater anomalies than those observed in the ocean. D) Cannot be determined by the information given.

14) What hypothesis best explains the reason for the observed differences in the data between the ocean and land?

A) B) C) D)

Land absorbs heat from the oceans. Water heats up and cools down faster than land. There is more ocean than land on Earth’s surface. Land heats up and cools down faster than water.

15) According to the second graph, which hemisphere is experiencing the greater temperature anomalies since the 1980s?

A) Northern B) Southern C) Both D) Neither

16) Which year marks the change from negative °C anomalies to positive °C anomalies?

A) 1880 B) 1900 C) 1940 D) 2000

Passage IV The developmental period for Drosophila melanogaster (fruit fly) varies with temperature, as with many ectothermic species. The shortest development time (egg to adult), 7 days, is achieved at 28 °C. Development times increase at higher temperatures (30 °C, 11 days) due to heat stress. Under ideal conditions, the development time is 8.5 days at 25 °C, 19 days at 18 °C and over 50 days at 12 °C. Under crowded conditions at 25 ˚C, development time increases to 12 days, and the emerging flies are smaller. Females lay some 400 eggs (embryos), about five at a time, into rotting fruit or other suitable material such as decaying mushrooms and sap fluxes. The eggs, which are about 0.5 mm long, hatch after 12-15 h (at 25 °C). The resulting larvae grow for about 4 days (at 25 °C) while molting twice (into 2nd- and 3rd-instar larvae), at about 24 and 48 h after eclosion. During this time, they feed on the microorganisms that decompose the fruit, as well as on the sugar of the fruit themselves. Then the larvae encapsulate in the puparium and undergo a four-day-long metamorphosis (at 25 °C), after which the adults emerge.

169

17) What trend best describes the information observed for the development of fruit flies over the temperature range between 12 °C and 28 °C?

A) As the temperature increases, the development time decreases. B) As the development time increases, the temperature decreases. C) As the temperature increases, the development time decreases. D) As crowding increases, the development time decreases.

18) About how long would it take for development to occur under extremely crowded conditions at 35 °C?

A) 5 days B) 8 days C) 11 days D) 18 days

19) A female fly lays 400 eggs, and half the eggs are allowed to develop at 12 °C while the other half are subjected to extreme crowding at 25 °C. What would you expect to observe in the two groups, based on their development conditions?

A) The second group to hatch would be extremely small. B) The first group would be normal sized. C) The two groups would hatch at the same time. D) The first group to hatch would be able to have offspring before the second group even hatched.

20) If a female fly that is smaller than normal due to development under crowded conditions were to lay eggs under ideal conditions, the flies that hatch would

A) be normal sized. B) be smaller than normal flies. C) take 12 days to develop. D) take 50 days to develop.

21) A female fly lays 400 eggs. The eggs initially begin their development under ideal conditions but are forced into crowded conditions for the remainder of development. What can be said about the size of the flies?

A) The flies will be smaller. B) The flies will be normal sized. C) Half will be normal sized and half will be smaller. D) There is not enough information to determine the outcome.

170

Passage V Isopods (pillbugs or woodlice) demonstrate negative photo taxis. They have light receptors that are sensitive to general illumination, and they make directed movements away from areas of greater illumination toward dark areas. Isopods are classic examples of animals which orient to humidity gradients via kinesis (more specifically, orthokinesis); that is, there is a change in the general level of movement with a change in the stimulus intensity. Stress that orients via kinesis is due to a nondirected locomotory activity. Isopods increase their movements under dry conditions and wander quite randomly, decreasing their activity under preferred humidity. Because of this decrease in activity in areas of high humidity, isopods tend to accumulate or aggregate in damp places. Taxis, on the other hand, is a direct orientation of an organism in response to a stimulus. Kinesis indicates a variation in intensity of locomotor activity that is dependent on the intensity of the stimulation, not its direction. These exercises demonstrate taxis and kinesis separately and in combination. Experiment 1 A moist paper was placed under a paper box (opaque box) at one end of a tray and a dry paper under a second paper box at the other end of the tray. The trays were illuminated as evenly as possible with an available light source. Ten isopods were then introduced to the center of the tray and allowed to wander for 30 minutes. The isopods were observed during this period, and the number of exits from each box was recorded. At the end of 30 minutes, the number of animals that were still wandering and the number under each box were recorded. It was observed that the isopods initially began moving and then began to aggregate under the box with the moist paper. Experiment 2 The experiment was set up as in the first experiment, but the tray was covered with aluminum foil so that the isopods were completely in the dark. It was observed that the isopods were moving less initially, but ultimately aggregated under the moist box. Experiment 3 A moist paper was placed under a plastic box (transparent box), and a dry paper was placed under a paper box. It was observed that the isopods began to aggregate around the paper box. Experiment 4 A dry paper was placed under both a plastic box and a paper box. It was observed that the isopods aggregated under the paper box.

171

22) In which experiment does humidity NOT play a factor?

A) Experiment 1 B) Experiment 3 C) Experiment 4 D) Experiment 2

23) Which experiment tests whether light or humidity is a stronger stimulus?

A) Experiment 2 B) Experiment 4 C) Experiment 1 D) Experiment 3

24) What should be observed if a moist paper were placed under both the plastic and paper boxes in the presence of light?

A) The isopods would aggregate under the plastic box. B) The isopods would aggregate under the paper box. C) The isopods will not move at all because of the high humidity. D) The isopods will aggregate evenly between the two boxes.

25) All of the conditions for response below are tested in the experiments EXCEPT:

A) light with no humidity difference B) light with humidity difference C) humidity with no light D) dark with no humidity difference

26) In what conditions do isopods prefer to exist?

A) low humidity and low light intensity B) high humidity and high light intensity C) high humidity and low light intensity D) low humidity and high light intensity

27) To which stimulus should the isopods have a stronger response?

A) Humidity B) Light C) Neither; the two elicit equivalent responses. D) More information is needed.

172

Passage VI Until recently, people simply knew that dinosaurs went extinct — their fossils were found throughout the Mesozoic era but were not located in the rock layers (strata) of the Cenozoic era. So, we knew that dinosaurs went extinct some 64-66 million years ago, but that was all. Many wild ideas about how the dinosaurs were rendered extinct were presented over the years. Few satisfactory answers to the mystery behind the extinction of dinosaurs were offered for a convincing mechanism for the “K-T extinction” (meaning the extinction of dinosaurs at the boundary between the Cretaceous period (K) and the Tertiary period (T)). However, it was noticed that at or near the K-T boundary (the layers of the earth mantle corresponding to this time frame) in several places around the globe, there exists a thin layer of clay with an unusually high iridium (a rare metal similar to platinum) content. Scientist 1 Major changes in the organization of the continental plates (continental drift) were occurring at the K-T boundary. This led to increased volcanism that could have created enough dust and soot to block out sunlight, producing the climatic change. In India during the Late Cretaceous period, huge volcanic eruptions were spewing forth floods of lava, which can be seen today at the K-T boundary (these ruptures in the Earth’s surface are called the Deccan traps). The chemical composition of the lava rocks in India shows that they originated in the Earth’s mantle, which is also relatively rich in iridium. This richness would explain the iridium layer. Scientist 2 The original hypothesis is the basis for several subsequent variations on the theme that a large extraterrestrial object collided with the Earth, its impact throwing up enough dust to cause the climatic change. The iridium layer is what proves that an asteroid impact could be responsible for the extinction — asteroids and similar extraterrestrial bodies are higher in iridium content than the Earth’s crust, so scientists figured that the iridium layer must be composed of the dust from the vaporized meteor. No crater was found, but it was assumed that one existed that was about 65 million years old and 100 kilometers (about 65 miles) in diameter. 28) Scientist 1 mentions all of the following as possible causes of dinosaur extinction EXCEPT:

A) volcanoes B) continental drift C) global climate change D) celestial bodies

173

29) A major difference between the theories of Scientist 1 and Scientist 2 is that Scientist 2 believes that A) a gradual climate change occurred, whereas Scientist 1 believes a rapid climate change occurred. B) an extraterrestrial catastrophe caused the climate change, whereas Scientist 1 believes continental drift was the cause for numerous volcanic eruptions. C) volcanoes are not powerful enough to cause a global climate change, whereas Scientist 1 believes that volcanoes are powerful enough. D) the iridium can only come from an extraterrestrial source, whereas Scientist 1 believes that asteroids do not contain enough iridium to account for the amount in the K-T boundary. 30) Which of the following, if true, could best defend Scientist 2’s theory against criticism voiced by Scientist 1? A) To date, no reliable evidence for any celestial bodies that may have crossed paths with Earth has been found. B) The fossil record suggests that several marine species died out millions of year prior to the K-T extinction. C) Celestial bodies, such as asteroids, contain higher concentrations of iridium than are present in the Earth’s mantle. D) Strong evidence exists for a series of volcanic eruptions during the K-T extinction. 31) Which discovery would support the hypothesis of Scientist 1? A) The presence of iridium was part of an unrelated phenomenom. B) Mass extinction occurred gradually over several million years. C) The K-T boundary was found to have millions of Deccan traps throughout various parts of the world. D) A massive crater was discovered recently on the Yucatan Peninsula in Mexico. 32) Both scientists agree that what was the underlying cause of the extinction of the dinosaurs?

A) A massive asteroid collided with the Earth, killing all the dinosaurs. B) A massive climate change resulted in the extinction of the dinosaurs. C) Volcanic eruptions resulted in the death of the dinosaurs. D) Iridium was the cause for the massive extinction.

174

33. The following statement supports which Scientist’s theory? “The marine fossil record does support a slightly rapid decline, while the terrestrial record (especially in North America) strongly suggests a more gradual decline….”

A) Scientist 1 B) Scientist 2 C) Both D) Neither

34) What data below could ultimately provide the best information to prove or disprove one of the arguments?

A) a more detailed astronomical history B) a more detailed fossil history C) an accurate mapping of continental drift D) all of the above

Passage VII A number of studies indicate that radon emissions occur from the earth. Some suggest that these emissions might serve as earthquake precursors. This study was undertaken to characterize the variability of radon emissions from the earth. The questions posed were: 1) Are there “bursts” or abrupt increases of radon gas? 2) Are these “bursts” random or periodic? 3) Are these abrupt increases distinguishable from the continual release of radiation from the earth? and 4) Is the sensor selected for this study suitable for this purpose? Experiment 1 A 20- to 22-inch hole with a 6-inch diameter was prepared in the ground. A RM-60 Microroentgen Radiation Monitor (Aware Electronics Corp., P.O. Box 4299, Wilmington, Delaware 19807) was placed in the hole about 6 inches from the bottom. Because this sensor is sensitive to radiation from a variety of sources, we compared its response under a number of conditions to show that the radon signal was distinguishable from other sources of background radiation. The sensor was placed with the window facing down, 6 inches above the bottom. Data were collected using Aware software.

175

Experiment 2 The sensor was suspended 5 feet above the ground in a small greenhouse close to the original hole described in experiment 1. It was observed that the counts diminished in amplitude to a very narrow band of 1100 to 1200 counts/hour (see figure below). No sign of periodicity was evident.

176

Experiment 3 Using the initial experimental setup with the sensor in the hole, on November 24, 1 inch of rain fell between 8 a.m. and 8 p.m. No clearly defined sign of periodicity remained. Interestingly, the range in the amplitude of the counts per hour remained essentially the same, even though the periodicity of the peak emissions was not evident.

35) What information can be learned from Experiment 2?

A) Heavy rain directly influences the signal. B) The periodicity was independent of the position of the sensor. C) The periodicity is directly related to the position of the sensor. D) The signal is independent of all outside factors.

36) It is well known that water-saturated soil restricts the exchange of gases between soil and the atmosphere. Thus, when soil gas is not being released under these conditions, observed radiation counts are reduced. Which experiment(s) would this information support?

A) Experiment 1 B) Experiment 2 C) Experiment 3 D) Experiments 2 and 3

37) The goal of which experiment was to answer each of the initially posed questions during its design?

A) Experiment 2 B) Experiment 1 C) Experiment 3 D) None

177

38) Which of the three experiments would be considered the control?

A) Experiment 1 B) Experiment 2 C) Experiment 3 D) There is no control.

39) The sensor was placed in a hole that was wet from rain. The soil then dried over the duration of the measurements. What result should be expected?

A) exactly the same as previously observed B) a signal showing a gradual recovery to periodicity C) a signal similar to that observed in Experiment 2 D) a signal similar to that observed in Experiment 1

40) What conclusion can be made based on the experiments?

A) Periodicity observed in radiation counts is due to radon emissions from soils. B) Periodicity observed in radiation count is due to other gas emissions from soils. C) Radon emissions do not display a periodic cycle of 24 hours. D) The experiments failed to answer the initially posed questions.

178

Chapter Seven: Answers to Practice Test Answers to Reading Test (Test may be found on Page 123.) 1. A. Although it seems as though it’s morning (bacon, coffee, waking up), context clues within the passage tell us it’s late in the day. Later in the passage, the father refers to the meal as supper. 2. B. The topic of conversation changes abruptly from the preceding paragraph to the following one. 3. A. You must read on further to determine how the boy feels here. He has tried so hard to prepare a meal and desperately wants his father to be pleased with it. The passage provides no evidence for the other choices. 4. D. Although the other choices all point to physical or emotional strain, a man need not be unhealthy to look silently at a lake. 5. A. The boy tries to laugh, but he feels sorrow because the entire meal is ruined. Also, he apologizes in the next paragraph. Choice C seems possible, but based on his attitude toward his father up to this point in the passage, we know that he feels comfortable around his father and just wants very much to please him. 6. B. It is necessary to look at the preceding paragraphs to understand the meaning of this line. The father holds back any comments that might hurt the boy’s feelings and “lightly” says he’s eager to eat some of his supper. The fact that he doesn’t eat much confirms that the comment is intended to spare the boy’s feelings. 7. C. Choices A and B are not metaphors, so they can be eliminated. Choice D, although a metaphor, does not refer to any type of struggle on the father’s part. Choice C provides an outward sign of his internal struggle. His face is a battleground as the battle is waged inside him. 8. C. When the boy is moved by the sunset, he plays his violin, which the author says is “no new thing.” 9. B. Plenty of foreshadowing throughout the passage suggests that something bad will happen. Although the boy is cheerful, the reader realizes that it is only his innocence that allows him to feel this way.

179

10. D. David shows his innocence in many ways throughout the passage, especially in his joy and optimism in spite of the obvious undercurrent of sadness. The father has clearly protected his son from sorrow to the best of his ability. 11. B. He says specifically that Europe is “rich in the accumulated treasures of age.” 12. A. In the first paragraph of the passage, the author speaks of the grand illusions he has of Europe. In the second paragraph, he mentions philosophical works that have led him to believe in the grandeur of Europe. 13. C. The word “baleful” is used today to mean evil or sinister, but in the early 1800s it meant miserable as well. A close look at the context clues in the passage should make the meaning clear. Remember, in this section of the test, you will be asked to figure out the meanings of words as they are used in passages. 14. C. Although the author uses all of these lines, only choice H does not include a poetic device. Choice F is a metaphor, choice G is personification, and choice J is an example of alliteration. 15. B. The author reasons that, if these little men are so important, others in their country must be truly great. Don’t be tricked by choice A. As readers, we detect arrogance in the Englishmen he speaks about, but the naive author is sincere, not sarcastic. 16. A. In the paragraph, he compares men, animals, and even landscapes, pointing out that those in Europe are far superior. 17. B. Although it’s good that he was able to travel, he’s frustrated by the fact that he returned home without seeing what he set out to see. 18. C. He has a passion (or desire) for roving (or traveling). Evidence in the passage supports this desire. 19. C. Rather than taking in the great sights, he enjoyed the small towns. Once he returns home, however, his heart almost fails him when he realizes that he missed out on the sights of the “regular traveller.”

180

20. B. The things the author saw in his travels may not be considered important by “regular travelers,” but that doesn’t make them any less great. The author must learn that it’s okay to appreciate things that others deem unimportant. 21. B. The article doesn’t address prevention or cures at all. Nor does the author suggest that parents should take their children to pediatric cardiologists. 22. C. The author points out that innocent is used more often because people often misunderstand the term functional, even though it means the same thing. 23. C. In the second paragraph, the author points out that murmurs do “not imply the presence of structural heart disease.” 24. B. Since a congenital heart defect happens during the development of the heart, it must happen in the first eight weeks of pregnancy. Don’t be fooled by choice F. Although the heart is developed by the end of eight weeks, the author never suggests that it’s fully functional. In other words, it cannot yet function on its own outside the womb. 25. B. The author states that less than 1% of babies are born with heart defects. 26. A. In the preceding paragraph, we learned that murmurs are most often heard in childhood; hence, they are common. The author has told us that they are not usually dangerous; he’s not addressing diagnosis, and he never mentions prevention. 27. B. The list of items constituting the focused history provides a clue for this answer. 28. D. The author states that some children are referred, but he makes no comment about whether or not they should be. 29. A. Although the pediatric cardiologist will ask about the child’s ability to engage in physical activity, it’s not a factor he sees during his physical examination. 30. D. We know that these sounds are abnormal because the author calls them “extra,” or beyond the normal sounds. They are not the sounds of valves closing, however; those are the “lub-dub” sounds.

181

31. A. An illusory feeling is one based on illusion, not on fact. Don’t be tricked by choice C just because the author mentions that point in the first paragraph. 32. B. The author states directly that walking is inherent, not culture dependent. 33. B. The author states that “there is every reason to believe that he will learn to walk” but that it is “certain that he will never learn to talk.” While the child may possibly create his own language, the author does not propose that idea in the passage. 34. B. Since walking is not influenced by culture, the child’s walking would be the same. Its speech, however, would be that of the new society. The author provides no evidence that the child would meet with any obstacles to language development. 35. C. Joy that would lead us to cry out would not be unspoken or slight. A sudden twinge of sudden joy would be redundant. 36. D. Although all of the choices may be true of speech, religion, and art, only choice J is addressed by the author. 37. D. The author argues repeatedly that these sounds cannot be considered speech. 38. B. Aloof means set apart. The author points out that these sounds don’t have standardized meanings. “Oh!” can mean something different in every situation. 39. C. Whereas the author doesn’t consider such an utterance to be speech, he does acknowledge it as instinctive, like walking. 40. B. Although he does compare walking and talking, he does so only to make his point that speech is not a natural instinct.

182

Answers to English Test (Test may be found on Page 138.) Passage 1 1. D. We need the semicolon to join the two independent clauses together. Also, I is the correct pronoun because it’s used as the subject of the second clause. 2. B. The correct verb is sat because there is no direct object. Although choice J uses sat as well, it adds a direct object (ourselves), making sat no longer correct. 3. C. The subject (not one) is singular, so it requires the verb has. Although choice D also uses has, it is unnecessarily wordy. 4. D. Slowly describes how the group started and must therefore be in its adverb form. Choice H uses slowly also, but it changes the two sentences into one comma splice. 5. B. The correct idiomatic expression is figured out. Choice C also uses figured out but is too wordy. 6. D. It is not necessary to use two coordinating conjunctions (and and so). We need the comma after great because we have a compound sentence. 7. B. The pronoun it refers to the antecedent catapults, so it must be changed to them. To give something a try is an idiomatic expression, so give them tries wouldn’t work. 8. D. Choices F and G are both passive voice. Choice H is unnecessarily wordy. 9. D. The correct word is who because it’s used as the subject of were watching. Which can be a subject as well, but it refers to things, not people. 10. D. Here we have a misplaced modifier. We weren’t flying 12 feet; the balls were. Choice G changes the sentence into a comma splice. Choice H suggests that the catapults were flying 12 feet. Although the correct answer isn’t usually the longest, choice J is an exception.

183

11. C. It’s important to be consistent with pronouns. Since the rest of the paragraph uses first person pronouns, this sentence should also. 12. C. Choices F and G both provide a sentence fragment beginning with Although. Choice J uses the wrong verb tense to be consistent with this paragraph. Choice H provides a past tense verb and combines the two sentences into one correctly punctuated sentence. 13. A. To make one team possessive, we add an apostrophe and an s after the word team. 14. D. Since the sentence compares two catapults, we must use the comparative form (-er) rather than the superlative form (-est). Also, farther, not further, refers to physical distance. 15. B. Choice B allows the sentence to be parallel (building catapults, reading a book). Choice C avoids the problem of parallel structure by adding a new clause, but the verb tense is wrong. It would have to be would have learned. Passage 2 1. B. The correct word for comparison is than. Choice C also uses than but is needlessly wordy. 2. C. It’s is a contraction meaning it is, which is correct in this sentence. You’re is a contraction meaning you are, which is not correct in this sentence. 3. D. The correct idiomatic expression is big man on top. 4. C. It is concise and corrects the comma splice. Hopefully, you’ve learned by now to recognize the excessively wordy choices like choice J. 5. C. The previous sentence talks about what to do until that day, so it makes sense for this sentence to address what to do for now. 6. B. The rest of the paragraph uses second person pronouns (like you), so we must be consistent here.

184

7. C. To make the sentence structure parallel, we must eliminate the extra subject (he). Don’t be fooled by D. It’s wordy and it’s passive voice. 8. D. Choices F and H both provide a sentence fragment starting with Because. Choice G turns the fragment into a comma splice. 9. B. It’s clear; it’s concise. Cut the extra words whenever possible. 10. B. Read the choices carefully. Choice J would work, but upperclassMEN has been changed to upperclassMAN. Choice F, by the way, is passive voice. Choice H uses they’re when it should be their. 11. D. Whew, look at those other wordy choices! 12. C. This one is all about pronoun/antecedent agreement. Upperclassmen agrees with their and with faces. Although the agreement is correct in choice G, look at the next sentence. The pronoun them indicates that you need the plural upperclassmen rather than the singular upperclassman. 13. A. Keep it short and simple. 14. C. The subject of the sentence is making fun (singular) and agrees with the verb causes. Choice J uses causes, but look at all those extra words! 15. C. In order to bring the paragraph full circle, it’s good to tie the closing line back in with the opening line. Choice C does just that without being wordy or awkward like the rest of the choices are. Passage 3 1. B. It’s clear and concise. It avoids the double negative of choice A, and it avoids the second person pronoun (you), which is not used anywhere else in the paragraph. 2. D. Keep it short. Choice F uses the wrong verb tense.

185

3. C. All of the other choices imply that the students can’t apply any of the concepts because they don’t understand any of them. Only choice C makes it clear that the concepts they can’t apply are only the ones that they don’t understand. 4. A. The structure is parallel and clear. The other choices just add confusion. 5. C. The lessons are more productive when students understand grammar. The conjunctions and and since both suggest that the students already do understand grammar. 6. C. The studies still show it. All of the other choices have unnecessary words. 7. C. Choice A has a subject/verb agreement problem. Choice B makes the sentence into a fragment. And certainly you recognized choice D as the wordy distracter. 8. B. Choices F and J present a subject/verb agreement problem. And the skills haven’t been important; they still are important. 9. B. Choice A is a comma splice. Choice C puts the semicolon on the wrong side of however. Finally, the and in choice D shows agreement rather than contrast. 10. D. The correct word is who because it acts as the subject of the verb participated. Which refers to things, not people. Although that can refer to groups of people, the unnecessary comma eliminates G as a viable choice. 11. D. By eliminating the underlined portion altogether, we create a simple yet effective appositive. 12. B. Choice F is passive voice. Although all of the other choices are active voice, G is the most clear and concise. 13. A. The paragraph is certainly not made-up or imaginary, so we can eliminate C and D. Since the author is trying to convince readers to agree with his opinion rather than provide an account of events, we can also eliminate B.

186

14. A. This sentence, when placed after the first sentence of the second paragraph, reinforces the impact that grammar instruction could have on this alarming statistic. 15. B. The passage has told readers that grammar instruction is important. Now this final sentence reinforces the reason it’s important. Choice C brings up a new topic; choice D introduces a first person pronoun not consistent with the rest of the passage; and choice A fails to bring closure to the paragraph. Passage 4 1. B. It’s the clearest and most concise choice. 2. D. The second sentence takes the idea of the first sentence one step further, making in fact an appropriate transition. Note that placing the word although at the beginning of this sentence would turn it into a fragment. 3. B. As it is, the sentence compares comments and plays. Choice B emphasizes the fact that we’re trying to compare comments in one play to comments in another play. Choice C won’t work because that is singular and therefore cannot refer to the plural comments. Choice D uses then (next) rather than than (a comparative word). 4. B. The conjunction for (rather than and) helps readers see the reason the satire is effective. Choice F is a comma splice, and choice H creates a fragment. 5. A. The semicolon is necessary because there are two independent clauses. Choice C places the semicolon on the wrong side of therefore, and choice D has too many conjunctions. 6. B. It’s active voice, and it’s clear. Choices F and J are passive voice. Although H is active voice as well, it’s awkward. 7. C. The helping verb have (creating present perfect tense) is necessary to be consistent with the rest of the paragraph.

187

8. C. Since hisself isn’t a word, we can eliminate choices F and J. You can also make himself possessive, so choice G doesn’t work either. 9. D. It is the only choice that makes the sentence parallel. 10. C. This question tests your understanding of subjunctive mood. With the words if and wish, we always use the verb were, even though it may sound strange. (Example: I wish I were rich!) 11. D. Use that to refer to groups of people as a whole. The semicolon (choice H) is not necessary, though, because we’re not joining two independent clauses. 12. C. Choice F is passive voice. Choice J is the wrong verb tense. Choice G would be acceptable, but the progressive tense is not necessary; therefore, go with the shorter choice. 13. B. Choice A is a sentence fragment. Choices C and D are needlessly wordy. 14. D. Oh, please do take the sentence out altogether! It doesn’t have anything to do with the content of the paragraph. 15. B. This sentence reinforces the content of the paragraph while bringing it full circle. Choices A and C are too broad and too narrow, respectively, and choice D merely restates the topic sentence. Passage 5 1. A. We use an apostrophe and an s after the word life to make it possessive. Don’t be fooled by the wordy choice C. 2. D. Funner is not a word; besides, it is not necessary to add an er when you use the word more. Choice G doesn’t work because it creates a subject/verb agreement problem. 3. B. Although choice C uses the right form of they’re, you don’t need all those words when one will do the job.

188

4. C. The semicolon is necessary to join the two independent clauses. Choices F and G are comma splices. Choice J would work if there were a comma after friend. 5. A. The line is supposed to be funny because it’s so obviously not true. 6. C. Use who when referring to people. Although that can be used to refer to a group of people, this sentence is really talking about individual friends. Besides, the comma is unnecessary. 7. B. None of the other choices makes the sentence parallel. 8. D. The sentence is redundant because it merely repeats the sentence before it. 9. D. Choice A is passive voice. Choices B and C are both active voice, but they create a pronoun/antecedent problem (them/friend). 10. A. This sentence is just fine as is. The other choices are awkward or wordy. 11. C. This sentence is intended to contrast with the one before it. Choice C offers the only contrasting transition. 12. B. The author is comparing two groups of characteristics, so these are more important rather than the most important. Choice J uses more, but it is a comma splice. 13. B. The singular subject (person) needs the verb is rather than are. Watch out for subjects that are widely separated from their verbs. They can be tricky. 14. A. The original wording is clear and concise. 15. C. The other choices are either awkward or too general.

189

Solutions to Mathematics Test (Test may be found on Page 153.) 1) D

x+y x+z ––––– = 5 ––––– = 8 2 2



x + y = 10 x + z = 16 y + z = 22 Solve for x; then substitute this expression in the next equation. x = 10 – y (10 – y) + z = 16 z=y+6 Substitute this value in the final equation. y + y + 6 = 22 2y + 6 =22 2y = 16 y=8 If y = 8, then 8 + z = 22. Thus, z = 14, choice D.

y+z ––––– = 11 2

2) C Since 16 is even, you must use the second relationship. Thus, 16 = ÷`√16 = 4. Since 9 is odd, 9 = 92 = 81. Therefore, 4 + 81 = 85. 3) A



3x + 7 =12 3x=5 x = 5/3 6 ( 5/3 ) – 5 = 10 – 5 = 5 Therefore, the answer is A.

4) E





30˚

80˚

60˚

Two angles of triangle ABC sum to 90°, meaning the remaining angle is 90°. By definition of intersecting lines, x = 90°.

190

5) D

Let “7 less than 4 times a number” = 4x – 7 Let “8 more than the number” = x + 8 Set the two equal and solve for x. 4x – 7 = x + 8 3x = 15 x=5

6) D The second graph represents the inverse of the original function (– x) followed by a translation of 3 units to the left (– x + 3). 7) B 8) E

The perimeter of the rectangle (p) = 1/5y + 1/5y + y + y = 12/5y To express y in terms of p, multiply both sides by 5/12 to give y = 5p/12. The answer is B. The area of A = 16; therefore, one side of A = 4. If one side of A = 4, then the perimeter of A = 4 + 4 + 4 + 4, or 16. Perimeter of A = 2/3 (perimeter of B) 16 = 2/3 (perimeter of B) Perimeter of B = 24 Perimeter of B = 2/3 (perimeter of C) 24 = 2/3 (perimeter of C) Perimeter of C = 36 So, one side of C must equal 36/4 = 9 Area of C = 92 = 81.

9) B Let x = total number of runs. x /12 = 6 runs per game for 12 games x = 72 total runs Now set up another relationship for the first 10 games. Let y = number of runs in the first 10 games. y /10 = 5 y = 50 runs in the first 10 games Number of runs in the last two games = (total runs) – (runs in the first 10 games) = 72 – 50 = 22 runs in the last two games If you’ve scored the same number of runs, you’re in the last two games, then 22/2 = 11 runs per game in each of the last two games.

191

10) C Factor the numerator: x2 – 2x + 1 = (x – 1)(x – 1) Examine the denominator: 1 – x = –1(x – 1) Solve the expression: (x – 1)(x – 1) ––––––––––– = – (x – 1) = 1 – x 1(x – 1) 11) C The easiest way to determine the area is to find the area of one triangle; and double the result (both triangles are equal). To find the area of one of the triangles, you need to find the third side. Using the Pythagorean theorem: a2 + 62 = 102 a2 + 36 = 100 a2 = 64 a=8 12) D 13) E

2x – 8y = 16 – 8y = – 2x +16 y = 1/4x – 2 Thus, m = 1/4 and the answer is J. Remember: distance = rate x time or d = rt Let distance traveled by Bill = db = 30t Let distance traveled by Tom = dt = 50t Thus, db + dt = 150 or 30t + 50t = 150 So t = 15/8 To find Tom’s distance: dt = 50t dt = 50(15/8) dt = 93.75

192

14) D equation of a line: y = mx + b –4 – 5 –9 m = –––––- = ––––– = 3/4 –3 – 9 –12

y = 3/4x + b Plug in the point (9,5) and solve for b: 5 = (3/4)(9) + b 5 = 27/4 + b b = – 7/4 Therefore: y = 3/4x + 27/4

15) B The perimeter of the rectangle (p) = 1/5y + 1/5y + y + y = 12/5y To express y in terms of p, multiply both sides by 5/12 to give y = 5p/12. The answer is B. 16) A



b + 2(x – 4) = s Solve for x. b + 2x – 8 = s 2x = s – b + 8 –b+8 x = s––––––– 2 s – b + 8 + 2 = ––––––––– s – b + 8 + 4/ = ––––––––– s – b + 12 x + 2 = ––––––––– 2 2 2 2

17) C

A is the midpoint of its side Let x = half the length of a side Let 2x = length of a full side Use the Pythagorean theorem to solve for x: x2+ (2x)2= 102 x2+ 4(x)2= 100 5(x)2= 100 x2= 20 x = 2√5 Full side length = 2x = 4√5 Area of square = side2 = (4√5)2 = 80.

193

18) B Set up the two inequalities and solve: 3x – 4 > 0 and 3x – 4 < 0 x > 4/3 and x < 4/3 19) C (s + 1)(s – 1) = s2 – 1 Since s2 = 19, s2 – 1 = 18 20) E Since f(a) = – b and f(– a) = b, the graph of the function passes through the origin (0,0), which happens to be the midpoint between the points (a, –b) and (–a, b). Thus, the x-intercept is the value where f(x) = 0. That value is x = 0, since it satisfies the function. 21) D Let Q = 2D D=N N = 3P, where Q= # of quarters, D = # of dimes, N = # of nickels, and P = # of pennies.



If Sarah has one quarter, then she has 2 dimes, 2 nickels, and 6 pennies. 1 Q = $0.25 2 D = $0.20 2 N = $0.10 6 P = $0.06 Total of the least amount of money Sarah can have: $0.61.

22) B ∆DCDA is proportionate to ∆DCBA. Since DA is a bisector, you know that /_DAC = 1/2 /_BAC The same relationship is true for –DCA and /_BCA ( /_DCA = 1/2 /_BCA). The relationship between /_ B and /_ D should be inversely proportional due to the fact that as D approaches B, /_D would decrease inversely to the other angles. Thus, /_B is 60˚ based on scaling /_D by 1/2. 23) E





Based on the triangle inequality rule, which states that the sum of the lengths of any two sides of a triangle is greater than the length of the third side, Base = 9 Let one of the other sides = x Then x + x > 9 2x > 9 x > 9/2 or 4.5 Since the sides are integers, the shortest possible side would be ~5. 194

24) B

Probability (red) = 1/4 out of 200 marbles = 50 red marbles Initial number of white marbles = 200 – 50 = 150 white marbles Addition of 50 white marbles gives a total of 200 white marbles Total number of marbles = 200 white + 50 red = 250 marbles Probability of selecting a white marble = 200/250 = 4/5

25) B By definition of intersecting lines, /_ a = /_ c; thus, you can eliminate any choice that has /_ c in it (choices A, C, and D. Also, –a is supplementary to /_d and /_b. Thus, /_b = /_d. By definition of parallel lines cut by a transversal, /_b = h = /_f. Thus b, d, h, and f must all be supplementary to /_a. The choice is therefore B. 26) A For the sum to add up to +74, you have to move from – 34 in the positive direction. Thus, the sum of all the negative even integers will be negative. You must have the same values on the positive end to cancel out the negative values. Thus, for a – 34 there must be a + 34. As a result, from – 34 to + 34, the sum is zero. The next two even positive integers are 36 and 38, whose sum is to 74.

27) A 5 + 2y ≥ 3 and 5 + 2y ≤ – 3 2y ≥ – 2 and 2y ≤ – 8 y ≥ – 1 and y ≤ – 4 28) B

Therefore, 2m = 38 m = 19

Prior to addition: Amount of cement = 1/2 (total amount) = 1/2 (12) = 6 tons Amount of granite and sand = 12 – 6 tons cement = 6 tons of granite and sand You want 60% of cement after adding x. After addition: Amount of cement = x + 6 Total amount of materials = x + 12



You need to set up a percent ratio to solve the problem: x+6 60 ––––– = –––– x + 12 100



Solve for x:

100x + 600 = 60x + 720 40x = 120 x = 3 tons of cement

195

29) C

You must use the distance formula to solve this problem. For all the line segments in the answers, you must find their lengths. d = √(x2 − x1)2 + (y2 − y1)2 AD = √(− 4 − 0)2 + (0 − 4)2 = √16 + 16 = √32 = 4√2 BD = √(− 2 − 0)2 + (0 − 4)2 = √4 + 16 = √20 = 2√5 AE = √(− 4 − 0)2 + (0 − (− 5))2 = √16 + 25 = √41 = √41 AC = √(− 4 − 2)2 + (0 − 0)2 = √36 + 0 = √36 = 6 CE = √(2 − 0)2 + (0 − (− 5)2 = √4 + 25 = √29



Thus, AE is the longest segment and the correct answer choice is C.

30) A First, find OM using the Pythagorean theorem. 42 = 22 + x2 16 = 4 + x2 12 = x2 x = √12 = 2√3 Since MO = ON, ON = 2√3.

Find MN using the Pythagorean theorem. MN2 = (2√3)2 + (2√3)2 MN2 = 12 + 12 = 24 MN = √24 = 2√6

31) C Plug y = –x into the other equation, then solve for x. (x – 5)2 + (– x – 5)2 = 4 x2 – 10x + 25 + x2 + 10x + 25 = 4 x2 – 50 = 4 x2 = 54 x = 3√6 and – 3√6 32) A The graph of [y] represent the graph in choice F. The equation given is equivalent to the equation x = [y] – 1, where the graph would be rotated 90° relative to the y-axis and then shifted to the left on the x-axis.

196

33) A Plug one equation into the other and solve for x. 10x – 3(3x – 1) = 9 10x – 9x + 3 = 9 x+3=9 x=6 Plug x = 6 into original equation and solve for y. y = 3(6) – 1 y = 17 The point is (6,17). 34) D Draw the triangle. Drop a perpendicular to the base.



Using trigonometric relationships:

Area of triangle = 1/2bh = 1/2 (12)(10 sin x) = 60 sin x 35) E



sin x = h/10 h = 10 sin x

2 16x - 1 = 64x (42)x2 - 1 = (43)x 42x2 - 2 = 43x 2x2 – 2 = 3x 2x2 – 3x – 2 = 0 Factor. (2x + 1)(x – 2) = 0 x = – 1/2 or x = 2

Because x = – 1/2, the answer is E.

197

36) B Definition for the reflection about the y-axis: for all points (x, f(x)), the reflected graph will contain points (–x, f(x)). Thus, a reflection about the y-axis is equivalent to translation of a point by 2x units, where the line y = 0 is the bisector of the distance between the original position and the new position. Thus, in the original graph, the vertex is at y = 4 and the graph is shifted right 2 units. In the reflected graph, the vertex occurs at y = 4 but the graph is shifted left 2 units to give the equation y = –3(x – 2)2 + 4, which is choice G. 37) C (x + y)2 = x2 + 2xy +y2 If 4xy = 8, then 2xy = 4 x2+ y2 + 2xy = 10 + 4 = 14 38) E If area (square) = 81, then each side of the square is 9. If perimeter of each ∆ = 30, then:

Perimeter of bold outlined figure = (sum of perimeters of all triangles) – (perimeter of square), since the dashed side of each triangle is a side of the square. Since there are four triangles, each with a dashed side, that equals the perimeter of the square.



The perimeter of the square is 9 + 9 + 9 + 9 = 36. Thus, 4 (30) – 36 = 120 – 36 = 84

39) D The only thing absolutely known about these two relationships is that they do not equal zero. Therefore, r, s, t, u, and v ≠ 0. The correct choice is D. 40) B Use the distance formula to first find the length of the radius.



d = √(x2 - x1)2 + (y2 - y1)2

√(- 6 - 0)2 + (- 4 - 2)2 = √36 + 36 = √72 = 6√2 Then use the formula with the center and each of the answer choices to find which choice has the same length. When you use this formula with (6,8): √(6 - 0)2 + (8 - 2)2 = √36 + 36 = √72 = 6√2 Thus G is the answer.

198

41) D 42) E





Let x represent number of seats in one section 300 seats ≤ x ≤ 400 seats Minimum number of total seats = 300 seats x 8 sections = 2400 seats Maximum number of total seats = 400 seats x 8 sections = 3200 seats Therefore, 2400 ≤ total number of seats ≤ 3200. The only choice that satisfies the inequality is 2600, choice D. Area of shaded region = area of circle – area of big square Area of individual square = x2 Side of individual square = x First, find the value of the diagonal in one of the small squares, since this is the radius of the circle. x2 + x2 = r2 2x2 = r2 Area of circle = πr2 = 2x2π Area of big square = 4x2 (4 small squares each with area x2) Area of shaded region = 2x2π – 4x2 = 2x2 (π– 2) The correct choice is K.

43) B Volume of cylinder = πr2h = π(2)2 (3) = 12π = ~39.68 To find number of full tanks to fill cylinder: # tanks = 39.68/3 = ~13.22 which means it takes over 13 full tanks 44) B

Points in 1st quarter: 1/8 of 48 = 6 Points in 2nd quarter: 1/3 of 48 = 16 Points in 3rd quarter: 1/4 of 48 = 12 Points in first 3 quarters = 6 + 16 + 12 = 34 Points in 4th quarter = total points – points in first 3 quarters = 48 – 34 = 14.

199

45) C



Let m = # of pounds 3.65 < m < 4.35 Because this is an inequality, you can eliminate choices A and B. Go to the remaining answer choices and solve the inequalities. [m - 4] < 0.35 Split into two inequalities and solve. m – 4 < 0.35 and m – 4 > – 0.35 m < 4.35 and m > 3.65 or 3.65 < m < 4.35

46) A If you drop a perpendicular down from point B to the x-axis, you will form a right triangle whose hypotenuse is 10 and whose base is 8. To find k, which is the length of the last leg, you will need the Pythagorean theorem. 102 = 82 + k2 100 = 64 + k2 k2 = 36 k=6 47) A Write out the 1st five terms: 1st term: 3 2nd term: 7 3rd term: –7 4th term: –3 5th term: 3 You are at the beginning of a loop in which, after every four terms, the 5th term gives the 1st term. To figure out any term: nthterm , ––––––– 4 where the remainder will determine which of the initial four values the term will take. For n = 155, 155/4 = 38 3/4, where the 3/4 represents that the loop has completed three of the four terms, or equals the 3rd term (– 7). 48) D Of Dwayne’s sales, he pays k/3 for lumber and supplies. That leaves 2k/3 left for savings. To save $5000, Dwayne must sell: 5000 15000 7500 –––––– = 5000 • 3/2k = –––––– = –––––– 2k /3 2k k

200

49) D



This problem involves an arithmetic series, so you need the equation for the sum of an arithmetic series: Sum of arithmetic series = n/2 (2a1 + (n – 1)d) You are given that the sum = 5250, the number of terms (n) = 12 and d (difference between terms) = 1. You will need to find the 1st term, a1, in order to find the 12th term. Plug the given values into the equation and solve for a1. 5250 = 12/2 (2a1 + (12 – 1)(1)) 5250 = 6(2a1 + 11) 875 = 2a1 + 11 2a1 = 864 a1 = 432 Next, you need the equation for the nth term of an arithmetic series: an = a1 + (n – 1)d Plug in the values n = 12, d = 1, and a1 = 432 to find a12. a12 = 432 + (12 – 1)(1) = 432 + 11 = 443.

50) D By definition of parallel lines cut by a transversal, x + y = 180 and x + x +130 = 180.



Solve for x.

2x + 130 = 180 2x = 50 x = 25 Plug x into the next equation to find y. x + y = 180 25 + y = 180 y = 155

51) A Start with x = 0. When x = 0, h(x) = 1. Therefore, 1 must be added somewhere in the equation. Look at the choices and eliminate B, C, and D because none of them involves adding 1. Plug x = 1 into the remaining equations to see which one satisfies the table.

201

52) C The area of the outer circle is 100π. To find the area of the inner circle, you need to find the length of its radius. The radius of the inner circle is the diagonal of the square. Use the Pythagorean theorem to find the length of the diagonal. The diagonal of a square forms two 45˚- 45˚- 90˚ right triangles, so the legs of the triangle formed with the square will be equal (4√2). (4√2)2+(4√2)2= c2 32 + 32 = c2 64 = c2 c=8 Thus, the area of the inner circle is 64π. One side of the square equals 4√2 + 4√2 = 8√2 The area of the square is (8√2)2= 128 Area of the shaded region (A) = (area of outer circle) – (area of inner circle) + (area of square) A = (100π – 64π) +128 = 36π + 128 53) E There are six faces to the prism. Within the six faces, there are three pairs of equal faces.

Area of first face of one pair: 6 x 8 = 48 x 2 faces = 96 Area of first face of second pair: 12 x 8 = 96 x 2 faces = 192 Area of first face of third pair: 6 x 12 = 72 x 2 faces = 144



Total surface area = 96 + 192 + 144 = 432. Thus, the answer is E.

54) A Draw ra ight triangle with sides corresponding to tan 0 = 4/3.



Find the hypotenuse using the Pythagorean theorem. 32 + 42 = hyp2 9 + 16 = hyp2 = 25 hyp = 5 Therefore, cos 0 = 3/5

202

55) D

56) D

If 1st flip = heads and last flip = heads, out of 14 flips, only a maximum of 14 - 1 flips that come up heads can be consecutive. The maximum number of consecutive heads must correspond to the maximum number of consecutive tails. Flips 1 – 13 = Heads (13 consecutive) 14 – 23 = Tails (10 consecutive) 24 = Heads



Let x and y represent the length and width of a rectangle, respectively. If you double those sides, you get 2x and 2y. The area of the initial figure is xy The area of the new figure is 2a x 2b = 4ab. To find the percentage of increase, set up a ratio: 4ab /ab = x/100 4 = x/100 x = 400 Percent increase = 400 – 100 = 300% increase

57) C

Volume of small cube = 2 x 2 x 2 = 8 Volume of large cube = 4 x 4 x 4 = 64 Twice volume of small cube = 16 Set up a ratio: 16 /64 = 1/4 full of water 1 /4 of height = 4 (1/4) = 1



58) D Let CD = x Let CE = ED = QE = PQ = x/2

2

Area of DPQE = 1/2 (QE)(PQ) = (1/2) (x/2)2 = x /8



Area of DPCD = 1/2 (CD)(PE)



PE2 = QE2 + PQ2





To find PE use Pythagorean theorem 2

= (x/2)2 + (x/2)2 = 2x2/4 = x /2 2



PE = √x /2 = x/√2 = x√2/2



Ratio of ∆PCD : ∆PQE = ∆PCD/∆PQE = x √2/4 / x /8 = x √2/4 • 8/x2 = 2√2/1 or 2√2 : 1.



2

Area of ∆PCD = 1/2 x(x√2/2) = x √2/4

2

2

203

2

59) E Use trigonometric identity: sin20 + cos20 = 1 Thus, sin20 = 1 – cos20 sin20 So ––––––– =1 1 - cos20 60) A

Use the distance formula to solve the question:



Then find which of the given points is not at a distance of 10 from the point (-1,0).





d = √(x2 - x1)2 + (y2 - y1)2

The only point that does not work is (-6,-7).

d = √(- 6 - (- 1))2 + (- 7 - 0)2 = √(- 5)2 - 72= √25 + 49 = √74 ≠ 10.

Solutions to Science Test (Test may be found on Page 173). 1. C 60 g of NH4Cl will dissolve in 100 g of water at 70 °C. So if you had 200 g of water at 70 °C, you could dissolve double the amount: 2 x 60 g = 120 g. 2. B For NaNO3, as temperature increases, the solubility increases. 3. C Look on the chart and find the temperature at which the KCl line and the NaCl line cross. That occurs at 34 °C. 4. A Look at the chart and find 40 °C. Trace upward along the 40 °C line until you reach the first line that appears in the choices. The first line you meet will be for NaCl. 5. C The only thing that directly affects the solubility according to the data in the chart is increasing the temperature. Solubility of a given compound is an innate property that changes with respect to temperature. 6. B In the introductory paragraph, it states “…lasers are available in the ultraviolet, infrared, and x-ray spectral regions.” Thus, lasers do not emit only visible light.

204

7. C An electron in a lowest energy state will not be able to drop any further to give off energy. Electrons in higher energy states can drop to lower levels to give off energy. Photons being emitted are by definition being accompanied by a release of energy. 8. C In experiment 1, a low-energy electron is excited to an excited state. In experiment 2, the electron lies in a higher energy state but spontaneously drops to a lower energy level. Thus, it began in a higher energy state. In experiment 3, an electron is excited by adding energy. Thus, the answer is experiments 1 and 3. 9. B Experiment 2 involves a spontaneous drop from higher level to a lower level. No energy was added for this to occur. 10. D We cannot say which offers the most promise because we do not know how much energy each experiment generates or consumes. 11. D The best way is to measure the amount of energy emitted. This will offer a critical gauge in determining which would be the most efficient laser. 12. B The observed trend is increasing temperature as time passes. 13. C The maximum observed anomalies for land are much higher than for the ocean. The ocean does not top anomalies at 0.6 above the zero line, whereas on land, the observed anomalies are approaching 1.2° C above the zero line. 14. D This is a hypothetical question having its basis in the science of heating and cooling. One should realize that it takes a lot longer to heat water than land. Thus, there will be a more gradual heating and cooling and fewer observed anomalies. 15. A The Northern Hemisphere is observing anomalies above the +0.5° C line, whereas the Southern Hemisphere is still below that line. 16. C Around 1940, there was a change to the net positive anomalies (above the zero line) from the previous net negative anomalies (below the zero line).

205

17. A Development time decreases as the temperature increases. At 12° C, development takes 50 days. At 28° C, development takes 7 days. 18. D Under a combination of heat stress (35° C) and crowding, you should expect the development time to be longer than both the development times for heat stress and crowding taken independently (11 and 12 days, respectively). Thus, the expected development time would be greater than 11 or 12. So, the answer is 18 days, or choice J. 19. D The group at 12 °C would take 50 days to develop, whereas the group under crowding at 25° C would develop in 12 days. Thus, the first group to hatch would be the small flies and the second group to develop would be normal flies. Given that there is a difference of 38 days between the development of the two groups, if members of the first group had any offspring, the offspring would be able to fully develop before the other group finished their development. 20. A The conditions under which the parent developed are irrelevant to the conditions in which its offspring develop. The size of the parent is due to environmental pressure, not genetics. 21. D The passage never states at what point during development that crowding plays a factor in the development. It could be the early stages or it could be the later stages. There just isn’t enough information to determine the outcome. 22. C Experiment 4 tests the response to light alone. Both papers being dry means that humidity has been removed as a variable. 23. D Experiment 3 does test humidity vs. light. Aggregation at the dry but opaque box means that the isopods would rather be shielded from light than exist in a moist environment. 24. B An experiment with two moist papers translates into a situation in which the humidity is very high. This means that the isopod movement will be slowed, but this won’t interfere with their desire to avoid the light. Thus, they will aggregate at the opaque paper box. 25. D None of the experiment’s tests dark with no humidity difference. Such an experiment would be set up in the dark with either two moist papers or two dry papers.

206

26. C The isopods avoid light but aggregate toward the moist paper in the absence of light. Thus, they prefer high humidity (moist paper) and low light intensity (opaque box). 27. B Experiment 3 shows that the response to light dominates over the response to humidity. 28. D Scientist 1’s argument revolves around a major climate change caused by factors on the planet, not by any celestial bodies. 29. B Scientist 2’s argument centers around a celestial body colliding with the planet, whereas Scientist 1’s argument centers on volcanic activity and continental drift as the underlying causes of the climate change. 30. C If celestial bodies are shown to have higher iridium concentrations, it could be used by Scientist 2 to address the discovery of the high iridium content at the K-T boundary. 31. C This discovery would mean that during that time period there were numerous volcanic eruptions, many of which could potentially occur at the same time, thus casting large amounts of ash, smoke, and other matter into the atmosphere. Scientist 1 could use this as evidence to support his argument. 32. B Both scientists agree on a massive climate change. They disagree on how that change came about. 33. A You would expect a more gradual decline in the scenario of Scientist 1. Continental drift occurred over a long period of time, so a gradual climate change could be expected. In Scientist 2’s scenario, if a celestial body hit the planet, one would expect a more rapid change to occur (immediate impact followed by the aftermath). 34. D All of the given data would give a better picture of what actually may have occurred. 35. C Experiment 2 shows that when the sensor is above the hole (not within it), no periodicity is observed. Thus, the position of the sensor directly relates to the observance of periodicity. 36. C Experiment 3 is the only experiment that involves moisture as a variable.

207

37. B Experiment 1 answered the initial question posed in the introductory paragraph. The experiment was properly able to measure the radon emissions separated from other emissions and observe periodicity. 38. A Experiment 1 was the basic experiment to prove the validity of the design; thus, it is the control. 39. B You should expect to see no periodicity initially (as in Experiment 2) but then begin to see a gradual recovery (as observed in Experiment 3) before a return to periodicity (as in Experiment 1). 40. A The overall experiment proves that the periodicity observed is due to the radon emissions. The sensor is designed to isolate radon emissions from all other emissions (stated in the introductory paragraph). Thus, if radon were not responsible for the periodicity, the overall experiment would be invalid because the experiment design would not answer the initially posed questions.

208

Chapter Eight: Additional Resources You may find these additional resources helpful as you prepare for the ACT and move toward college planning: • Club Z! Tutoring (www.ClubZ.com) • The Hipp List by IVY1600, Inc. (www.ivy1600.com) • Tooth and Nail: A Novel Approach, by Charles Harrington Elster and Joseph Elliott • American College Testing (www.act.com) • My College Options (www.mycollegeoptions.com) • The National Association for College Admission Counseling (www.nacacnet.org) • The Road to College: A High School Student’s Guide to Discovering Your Passion, Getting Involved, and Getting Admitted, by Joyce E. Suber and the Staff of The Princeton Review

209

210

About the Authors Dawn Burnette, author of the verbal section of The Essential Guide to the ACT, is a National Board Certified Teacher who has taught high school English for fifteen years. She is also certified by the State of Georgia in gifted education for grades K-12. Dawn holds a B.A. in English Education and Journalism from Lenoir-Rhyne College and an M.A. in English Education from Georgia State University. In addition to her work on this guide, Dawn has published several works, including Daily Grammar Practice (a program for helping students in first grade through college to understand, remember, and apply grammar concepts); The Burnette Writing Process (an individualized, web-supported writing approach for grades 6-12); Vocabulary: A Novel Solution (a literature-based vocabulary program); and DGP Plus: Building Stronger Writers (writing strategies for helping students transfer grammar concepts to writing). A finalist for 2007 Georgia Teacher of the Year and recipient of a National Council of Teachers of English Teacher of Excellence Award, Dawn has spoken at conferences and to school systems all around the country. Stefan France, author of the mathematics and science sections of The Essential Guide to the ACT, graduated from Duke University in 2000 with a B.S. in Chemistry. Stefan has earned numerous academic awards for his outstanding scholarship and abilities in math and science. After obtaining his Ph.D. in Chemistry from The Johns Hopkins University in 2005, he joined Emory University as a postdoctoral associate. In 2007, he joined the faculty at Georgia Institute of Technology as an assistant professor of chemistry. He has also served as a math tutor for middle and high school students at ClubZ!. Joyce Suber, author and editor of The Essential Guide to the ACT, is a seasoned educator. A graduate of the University of Illinois, she holds a B.A. degree in Sociology and earned a 4.0 in the graduate Educational Psychology program at the National College of Education (National-Louis University). Joyce is certified by the State of Illinois to teach grades K-12, with concentrations in Language Arts and Social Studies (grades 6-12), and has taught high school English in both public and independent schools. In 2007, Joyce completed a book published by The Princeton Review (Random House) entitled Road to College: A High School Student’s Guide to Discovering Your Passion, Getting Involved, and Getting Admitted. She has served on numerous boards and committees of professional organizations in the field of college admission counseling. She has also spoken at numerous conferences and schools throughout the United States.

211

Co-Authors: The Student Perspective Diane Darling is currently in the 11th grade, but she is attending the University of West Georgia full-time in a dual enrollment program, taking college courses while earning her high school diploma. She currently has a 4.0 GPA and consistently scores in the top 95% of her class on both the SAT and ACT exams. Erin Gard is currently in the 11th grade and is on track to graduate early and begin her college career. She scored in the top 98% of her class on the ACT exam.

212

The Essential Guide to the ACT - Practice Test Name: ___________________________ Reading Test (Test on page 123) 1. A 2. A 3. A 4. A 5. A 6. A 7. A 8. A 9. A 10. A 11. A 12. A 13. A 14. A 15. A 16. A 17. A 18. A 19. A 20. A 21. A 22. A 23. A 24. A 25. A 26. A 27. A 28. A 29. A 30. A 31. A 32. A 33. A 34. A 35. A 36. A 37. A 38. A 39. A 40. A

B B B B B B B B B B B B B B B B B B B B B B B B B B B B B B B B B B B B B B B B

C C C C C C C C C C C C C C C C C C C C C C C C C C C C C C C C C C C C C C C C

English Test Passage 1 (p. 138) D D D D D D D D D D D D D D D D D D D D D D D D D D D D D D D D D D D D D D D D

E E E E E E E E E E E E E E E E E E E E E E E E E E E E E E E E E E E E E E E E

1. 2. 3. 4. 5. 6. 7. 8. 9. 10. 11. 12. 13. 14. 15.

213

A A A A A A A A A A A A A A A

B B B B B B B B B B B B B B B

C C C C C C C C C C C C C C C

D D D D D D D D D D D D D D D

E E E E E E E E E E E E E E E

214

The Essential Guide to the ACT - Practice Test Name: ___________________________ English Test Passage 2 (p. 141) 1. 2. 3. 4. 5. 6. 7. 8. 9. 10. 11. 12. 13. 14. 15.

A A A A A A A A A A A A A A A

B B B B B B B B B B B B B B B

C C C C C C C C C C C C C C C

English Test Passage 4 (p. 147)

English Test Passage 3 (p. 144) D D D D D D D D D D D D D D D

E E E E E E E E E E E E E E E

1. 2. 3. 4. 5. 6. 7. 8. 9. 10. 11. 12. 13. 14. 15.

A A A A A A A A A A A A A A A

B B B B B B B B B B B B B B B

C C C C C C C C C C C C C C C

215

D D D D D D D D D D D D D D D

E E E E E E E E E E E E E E E

1. 2. 3. 4. 5. 6. 7. 8. 9. 10. 11. 12. 13. 14. 15.

A A A A A A A A A A A A A A A

B B B B B B B B B B B B B B B

C C C C C C C C C C C C C C C

D D D D D D D D D D D D D D D

E E E E E E E E E E E E E E E

216

The Essential Guide to the ACT - Practice Test Name: ___________________________ English Test Passage 5 (p. 150) 1. 2. 3. 4. 5. 6. 7. 8. 9. 10. 11. 12. 13. 14. 15.

A A A A A A A A A A A A A A A

B B B B B B B B B B B B B B B

C C C C C C C C C C C C C C C

Mathematics Test (Test on page 153) D D D D D D D D D D D D D D D

E E E E E E E E E E E E E E E

1. A 2. A 3. A 4. A 5. A 6. A 7. A 8. A 9. A 10. A 11. A 12. A 13. A 14. A 15. A 16. A 17. A 18. A 19. A 20. A 21. A 22. A 23. A 24. A 25. A 26. A 27. A 28. A 29. A 30. A

B B B B B B B B B B B B B B B B B B B B B B B B B B B B B B

C C C C C C C C C C C C C C C C C C C C C C C C C C C C C C

217

D D D D D D D D D D D D D D D D D D D D D D D D D D D D D D

E E E E E E E E E E E E E E E E E E E E E E E E E E E E E E

31. A 32. A 33. A 34. A 35. A 36. A 37. A 38. A 39. A 40. A 41. A 42. A 43. A 44. A 45. A 46. A 47. A 48. A 49. A 50. A 51. A 52. A 53. A 54. A 25. A 56. A 57. A 58. A 59. A 60. A

B B B B B B B B B B B B B B B B B B B B B B B B B B B B B B

C C C C C C C C C C C C C C C C C C C C C C C C C C C C C C

D D D D D D D D D D D D D D D D D D D D D D D D D D D D D D

E E E E E E E E E E E E E E E E E E E E E E E E E E E E E E

218

The Essential Guide to the ACT - Practice Test Name: ___________________________ Science Test (Test on page 173) 1. A 2. A 3. A 4. A 5. A 6. A 7. A 8. A 9. A 10. A 11. A 12. A 13. A 14. A 15. A 16. A 17. A 18. A 19. A 20. A 21. A 22. A 23. A 24. A 25. A

B B B B B B B B B B B B B B B B B B B B B B B B B

C C C C C C C C C C C C C C C C C C C C C C C C C

D D D D D D D D D D D D D D D D D D D D D D D D D

E E E E E E E E E E E E E E E E E E E E E E E E E

26. 27. 28. 29. 30. 31. 32. 33. 34. 35. 36. 37. 38. 39. 40.

A A A A A A A A A A A A A A A

B B B B B B B B B B B B B B B

C C C C C C C C C C C C C C C

219

D D D D D D D D D D D D D D D

E E E E E E E E E E E E E E E

220

221

222

223

224